Evidence PDF
Evidence PDF
Evidence PDF
EVIDENCE i.
EVIDENCE
TABLE OF CONTENTS
I. GENERAL CONSIDERATIONS . . . . . . . . . . . . . . . . . . . . . . . . . . . . . . .1
A. DEFINING EVIDENCE LAW . . . . . . . . . . . . . . . . . . . . . . . . . . . . .1
B. SOURCES OF EVIDENCE LAW . . . . . . . . . . . . . . . . . . . . . . . . . . . .1
C. FEDERAL RULES OF EVIDENCE . . . . . . . . . . . . . . . . . . . . . . . . . .1
D. THRESHOLD ADMISSIBILITY ISSUES . . . . . . . . . . . . . . . . . . . . . . .1
1. Materiality: The Proposition to Be Proved . . . . . . . . . . . . . . . . . . . .1
2. Relevance: Probativeness—The Link Between Proof and Proposition . . . . .1
3. Federal Rules—Materiality and Probativeness Combined . . . . . . . . . . . .2
4. Competence . . . . . . . . . . . . . . . . . . . . . . . . . . . . . . . . . . . . .2
a. Policies Related to Truth-Seeking . . . . . . . . . . . . . . . . . . . . . . .2
b. Policies External to Litigation . . . . . . . . . . . . . . . . . . . . . . . . .2
E. EVIDENCE CLASSIFICATIONS . . . . . . . . . . . . . . . . . . . . . . . . . . .2
1. Direct or Circumstantial . . . . . . . . . . . . . . . . . . . . . . . . . . . . . .2
a. Direct Evidence Relies on Actual Knowledge . . . . . . . . . . . . . . . . .2
b. Circumstantial Evidence Relies on Inference . . . . . . . . . . . . . . . . .2
2. Testimonial, Documentary, or Real . . . . . . . . . . . . . . . . . . . . . . . .3
F. LIMITED ADMISSIBILITY . . . . . . . . . . . . . . . . . . . . . . . . . . . . . .3
1. Admissible for One Purpose But Not Another . . . . . . . . . . . . . . . . . .3
2. Admissible Against One Party But Not Another . . . . . . . . . . . . . . . . .3
3. Jury Must Be Properly Instructed . . . . . . . . . . . . . . . . . . . . . . . . .3
II. RELEVANCE . . . . . . . . . . . . . . . . . . . . . . . . . . . . . . . . . . . . . . . . .3
A. INTRODUCTION . . . . . . . . . . . . . . . . . . . . . . . . . . . . . . . . . . . .3
B. DETERMINING RELEVANCE . . . . . . . . . . . . . . . . . . . . . . . . . . . .4
1. General Rule—Must Relate to Time, Event, or Person in Controversy . . . . .4
2. Exceptions—Certain Similar Occurrences Are Relevant . . . . . . . . . . . .4
a. Causation . . . . . . . . . . . . . . . . . . . . . . . . . . . . . . . . . . . .5
b. Prior False Claims or Same Bodily Injury . . . . . . . . . . . . . . . . . .5
c. Similar Accidents or Injuries Caused by Same Event or Condition . . . . .5
1) Absence of Similar Accidents . . . . . . . . . . . . . . . . . . . . . . .5
d. Previous Similar Acts Admissible to Prove Intent . . . . . . . . . . . . . .5
e. Rebutting Claim of Impossibility . . . . . . . . . . . . . . . . . . . . . . .6
f. Sales of Similar Property . . . . . . . . . . . . . . . . . . . . . . . . . . .6
g. Habit . . . . . . . . . . . . . . . . . . . . . . . . . . . . . . . . . . . . . .6
h. Industrial or Business Routine . . . . . . . . . . . . . . . . . . . . . . . .6
i. Industrial Custom as Evidence of Standard of Care . . . . . . . . . . . . .7
1) Industrial Custom Distinguished from Business Routine . . . . . . .7
2) Relevant to Standard of Care But Not Conclusive . . . . . . . . . . .7
C. DISCRETIONARY EXCLUSION OF RELEVANT EVIDENCE (PRAGMATIC
RELEVANCE) . . . . . . . . . . . . . . . . . . . . . . . . . . . . . . . . . . . . . .7
D. EXCLUSION OF RELEVANT EVIDENCE FOR PUBLIC POLICY REASONS
(POLICY-BASED RELEVANCE) . . . . . . . . . . . . . . . . . . . . . . . . . . .7
1. Liability Insurance . . . . . . . . . . . . . . . . . . . . . . . . . . . . . . . . .7
ii. EVIDENCE
4. Exhibition of Injuries . . . . . . . . . . . . . . . . . . . . . . . . . . . . . . . 21
5. Jury View of the Scene . . . . . . . . . . . . . . . . . . . . . . . . . . . . . . 21
6. Demonstrations . . . . . . . . . . . . . . . . . . . . . . . . . . . . . . . . . . 21
a. Demonstrations Showing Effect of Bodily Injury . . . . . . . . . . . . . 21
b. Demonstrations Under Sole Control of Witness Are Excluded . . . . . . 22
c. Scientific Experiments . . . . . . . . . . . . . . . . . . . . . . . . . . . . 22
V. DOCUMENTARY EVIDENCE . . . . . . . . . . . . . . . . . . . . . . . . . . . . . . . 22
A. IN GENERAL . . . . . . . . . . . . . . . . . . . . . . . . . . . . . . . . . . . . . . 22
B. AUTHENTICATION . . . . . . . . . . . . . . . . . . . . . . . . . . . . . . . . . . 22
1. Quantum of Proof of Authenticity . . . . . . . . . . . . . . . . . . . . . . . . 22
2. Authentication by Pleadings or Stipulation . . . . . . . . . . . . . . . . . . . 22
3. Evidence of Authenticity . . . . . . . . . . . . . . . . . . . . . . . . . . . . . 22
a. Admissions . . . . . . . . . . . . . . . . . . . . . . . . . . . . . . . . . . 23
b. Testimony of Eyewitness . . . . . . . . . . . . . . . . . . . . . . . . . . . 23
c. Handwriting Verifications . . . . . . . . . . . . . . . . . . . . . . . . . . 23
1) Nonexpert Opinion . . . . . . . . . . . . . . . . . . . . . . . . . . . 23
2) Comparison of Writings . . . . . . . . . . . . . . . . . . . . . . . . 23
d. Ancient Documents . . . . . . . . . . . . . . . . . . . . . . . . . . . . . . 23
1) Federal Rules Distinguished from Majority of Jurisdictions . . . . . 23
e. Reply Letter Doctrine . . . . . . . . . . . . . . . . . . . . . . . . . . . . 24
f. Circumstantial Evidence in General . . . . . . . . . . . . . . . . . . . . 24
g. Photographs . . . . . . . . . . . . . . . . . . . . . . . . . . . . . . . . . . 24
1) Unattended Camera—Proper Operation of Camera . . . . . . . . . 24
h. X-Ray Pictures, Electrocardiograms, Etc. . . . . . . . . . . . . . . . . . 24
4. Compare—Authentication of Oral Statements . . . . . . . . . . . . . . . . . 24
a. When Necessary . . . . . . . . . . . . . . . . . . . . . . . . . . . . . . . 24
b. Methods of Authentication . . . . . . . . . . . . . . . . . . . . . . . . . . 25
1) Voice Identification . . . . . . . . . . . . . . . . . . . . . . . . . . . 25
2) Telephone Conversations . . . . . . . . . . . . . . . . . . . . . . . . 25
5. Self-Authenticating Documents . . . . . . . . . . . . . . . . . . . . . . . . . . 25
C. BEST EVIDENCE RULE . . . . . . . . . . . . . . . . . . . . . . . . . . . . . . . 26
1. Rule Expresses Preference for Originals . . . . . . . . . . . . . . . . . . . . . 26
2. Applicability of the Rule . . . . . . . . . . . . . . . . . . . . . . . . . . . . . 26
3. Nonapplicability of the Rule . . . . . . . . . . . . . . . . . . . . . . . . . . . 26
a. Fact to Be Proved Exists Independently of Any Writing . . . . . . . . . . 26
b. Writing Is Collateral to Litigated Issue . . . . . . . . . . . . . . . . . . . 27
c. Summaries of Voluminous Records . . . . . . . . . . . . . . . . . . . . . 27
d. Public Records . . . . . . . . . . . . . . . . . . . . . . . . . . . . . . . . 27
4. Definitions of “Writings,” “Original,” and “Duplicate” . . . . . . . . . . . . 28
a. Writings, Recordings, and Photographs . . . . . . . . . . . . . . . . . . 28
b. “Original” . . . . . . . . . . . . . . . . . . . . . . . . . . . . . . . . . . . 28
c. Admissibility of Duplicates . . . . . . . . . . . . . . . . . . . . . . . . . 28
5. Admissibility of Secondary Evidence of Contents . . . . . . . . . . . . . . . . 28
a. Satisfactory Foundation . . . . . . . . . . . . . . . . . . . . . . . . . . . 28
1) Loss or Destruction of Original . . . . . . . . . . . . . . . . . . . . 28
2) Original Outside Jurisdiction and Unobtainable . . . . . . . . . . . 28
EVIDENCE v.
a. Misleading . . . . . . . . . . . . . . . . . . . . . . . . . . . . . . . . . . 36
b. Compound . . . . . . . . . . . . . . . . . . . . . . . . . . . . . . . . . . 36
c. Argumentative . . . . . . . . . . . . . . . . . . . . . . . . . . . . . . . . 36
d. Conclusionary . . . . . . . . . . . . . . . . . . . . . . . . . . . . . . . . 36
e. Assuming Facts Not in Evidence . . . . . . . . . . . . . . . . . . . . . . 36
f. Cumulative . . . . . . . . . . . . . . . . . . . . . . . . . . . . . . . . . . 36
g. Harassing or Embarrassing . . . . . . . . . . . . . . . . . . . . . . . . . 37
h. Calls for a Narrative Answer . . . . . . . . . . . . . . . . . . . . . . . . 37
i. Calls for Speculation . . . . . . . . . . . . . . . . . . . . . . . . . . . . . 37
j. Lack of Foundation . . . . . . . . . . . . . . . . . . . . . . . . . . . . . . 37
k. Nonresponsive Answer . . . . . . . . . . . . . . . . . . . . . . . . . . . . 37
3. Use of Memoranda by Witness . . . . . . . . . . . . . . . . . . . . . . . . . . 37
a. Present Recollection Revived—Refreshing Recollection . . . . . . . . . . 37
b. Past Recollection Recorded—Recorded Recollection . . . . . . . . . . . 37
c. Inspection and Use in Cross-Examination . . . . . . . . . . . . . . . . . 38
C. OPINION TESTIMONY . . . . . . . . . . . . . . . . . . . . . . . . . . . . . . . . 38
1. Opinion Testimony by Lay Witnesses . . . . . . . . . . . . . . . . . . . . . . 38
a. General Rule of Inadmissibility . . . . . . . . . . . . . . . . . . . . . . . 38
b. When Admissible . . . . . . . . . . . . . . . . . . . . . . . . . . . . . . . 39
c. Procedure . . . . . . . . . . . . . . . . . . . . . . . . . . . . . . . . . . . 39
d. Situations Where Opinions of Lay Witnesses Are Admissible . . . . . . . 39
1) General Appearance or Condition of a Person . . . . . . . . . . . . 39
2) State of Emotion . . . . . . . . . . . . . . . . . . . . . . . . . . . . . 39
3) Matters Involving Sense Recognition . . . . . . . . . . . . . . . . . 39
4) Voice or Handwriting Identification . . . . . . . . . . . . . . . . . . 40
5) Speed of Moving Object . . . . . . . . . . . . . . . . . . . . . . . . 40
6) Value of Own Services . . . . . . . . . . . . . . . . . . . . . . . . . 40
7) Rational or Irrational Nature of Another’s Conduct (Sanity) . . . . 40
8) Intoxication . . . . . . . . . . . . . . . . . . . . . . . . . . . . . . . 40
e. Situations Where Opinions of Lay Witnesses Are Not Admissible . . . . 40
1) Agency or Authorization . . . . . . . . . . . . . . . . . . . . . . . . 40
2) Contract or Agreement . . . . . . . . . . . . . . . . . . . . . . . . . 40
2. Opinion Testimony by Expert Witnesses . . . . . . . . . . . . . . . . . . . . 40
a. Requirements of Expert Testimony . . . . . . . . . . . . . . . . . . . . . 40
1) Subject Matter Must Be Appropriate for Expert Testimony . . . . 41
2) Witness Must Be Qualified as an Expert . . . . . . . . . . . . . . . 41
3) Expert Must Possess Reasonable Probability Regarding His
Opinion . . . . . . . . . . . . . . . . . . . . . . . . . . . . . . . . . 41
4) Opinion Must Be Supported by Proper Factual Basis . . . . . . . . 41
a) Personal Observation . . . . . . . . . . . . . . . . . . . . . . . 41
b) Facts Made Known to Expert at Trial . . . . . . . . . . . . . . 42
c) Facts Made Known to Expert Outside Court . . . . . . . . . . 42
b. Opinion May Embrace Ultimate Issue . . . . . . . . . . . . . . . . . . . 42
1) Exception—Criminal Defendant’s Mental State . . . . . . . . . . . 42
c. Authoritative Texts and Treatises . . . . . . . . . . . . . . . . . . . . . . 42
D. CROSS-EXAMINATION . . . . . . . . . . . . . . . . . . . . . . . . . . . . . . . 43
1. Necessity for Cross-Examination . . . . . . . . . . . . . . . . . . . . . . . . . 43
2. Scope of Cross-Examination . . . . . . . . . . . . . . . . . . . . . . . . . . . 43
EVIDENCE vii.
a. Restrictions on Scope . . . . . . . . . . . . . . . . . . . . . . . . . . . . . 43
b. Significance of Restrictions . . . . . . . . . . . . . . . . . . . . . . . . . 43
c. Collateral Matters . . . . . . . . . . . . . . . . . . . . . . . . . . . . . . 44
E. CREDIBILITY—IMPEACHMENT . . . . . . . . . . . . . . . . . . . . . . . . . 44
1. Accrediting or Bolstering . . . . . . . . . . . . . . . . . . . . . . . . . . . . . 44
a. General Rule—No Bolstering Until Witness Impeached . . . . . . . . . 44
b. Exceptions . . . . . . . . . . . . . . . . . . . . . . . . . . . . . . . . . . . 44
1) Timely Complaint . . . . . . . . . . . . . . . . . . . . . . . . . . . . 44
2) Prior Identification . . . . . . . . . . . . . . . . . . . . . . . . . . . 45
2. Any Party May Impeach . . . . . . . . . . . . . . . . . . . . . . . . . . . . . 45
3. Impeachment Methods—Cross-Examination and Extrinsic Evidence . . . . 45
a. Prior Inconsistent Statements . . . . . . . . . . . . . . . . . . . . . . . . 45
1) Laying a Foundation . . . . . . . . . . . . . . . . . . . . . . . . . . 45
2) Evidentiary Effect of Prior Inconsistent Statements . . . . . . . . . 46
b. Bias or Interest . . . . . . . . . . . . . . . . . . . . . . . . . . . . . . . . 46
1) Foundation . . . . . . . . . . . . . . . . . . . . . . . . . . . . . . . . 46
2) Justification for Bias . . . . . . . . . . . . . . . . . . . . . . . . . . 46
c. Conviction of Crime . . . . . . . . . . . . . . . . . . . . . . . . . . . . . 46
1) Actual Conviction Required . . . . . . . . . . . . . . . . . . . . . . 47
2) Type of Crime . . . . . . . . . . . . . . . . . . . . . . . . . . . . . . 47
a) Crime Involving Dishonesty or False Statement . . . . . . . . . 47
b) Felony Not Involving Dishonesty or False Statement . . . . . . 47
(1) Accused in Criminal Case . . . . . . . . . . . . . . . . . . 47
(2) Witness Other than Accused in Criminal Case . . . . . . 47
(3) Compare the Balancing Tests . . . . . . . . . . . . . . . . 47
3) Must Not Be Too Remote . . . . . . . . . . . . . . . . . . . . . . . . 48
4) Juvenile Adjudication Generally Not Admissible . . . . . . . . . . . 48
5) Effect of Pardon Depends on Basis . . . . . . . . . . . . . . . . . . . 48
6) Pending Appeal Does Not Affect Admissibility . . . . . . . . . . . . 48
7) Constitutionally Defective Conviction Invalid for All Purposes . . . 48
8) Means of Proof—Extrinsic Evidence Permitted . . . . . . . . . . . 48
d. Specific Instances of Misconduct—Bad Acts . . . . . . . . . . . . . . . . 48
1) General Rule—Interrogation Permitted . . . . . . . . . . . . . . . . 48
2) Counsel Must Inquire in Good Faith . . . . . . . . . . . . . . . . . 49
3) Extrinsic Evidence Not Permitted . . . . . . . . . . . . . . . . . . . 49
4) Cannot Reference Consequences of Bad Act . . . . . . . . . . . . . 49
e. Opinion or Reputation Evidence for Truthfulness . . . . . . . . . . . . . 49
1) By Proof of Reputation . . . . . . . . . . . . . . . . . . . . . . . . . 49
2) By Opinion Evidence . . . . . . . . . . . . . . . . . . . . . . . . . . 49
f. Sensory Deficiencies . . . . . . . . . . . . . . . . . . . . . . . . . . . . . 49
1) Defects of Capacity . . . . . . . . . . . . . . . . . . . . . . . . . . . 50
a) Perceptive Disabilities . . . . . . . . . . . . . . . . . . . . . . . 50
b) Lack of Memory . . . . . . . . . . . . . . . . . . . . . . . . . . 50
c) Mental Disorders . . . . . . . . . . . . . . . . . . . . . . . . . 50
2) Lack of Knowledge . . . . . . . . . . . . . . . . . . . . . . . . . . . 50
a) Expert Witnesses . . . . . . . . . . . . . . . . . . . . . . . . . 50
b) Opinion Witnesses . . . . . . . . . . . . . . . . . . . . . . . . . 50
c) Character Witnesses . . . . . . . . . . . . . . . . . . . . . . . . 50
viii. EVIDENCE
g. Contradictory Facts . . . . . . . . . . . . . . . . . . . . . . . . . . . . . 51
4.Impeachment on Collateral Matter . . . . . . . . . . . . . . . . . . . . . . . 51
5.Impeachment of Hearsay Declarant . . . . . . . . . . . . . . . . . . . . . . . 51
6.Rehabilitation . . . . . . . . . . . . . . . . . . . . . . . . . . . . . . . . . . . 52
a. Explanation on Redirect . . . . . . . . . . . . . . . . . . . . . . . . . . . 52
b. Good Reputation for Truthfulness . . . . . . . . . . . . . . . . . . . . . 52
c. Prior Consistent Statement . . . . . . . . . . . . . . . . . . . . . . . . . 52
1) Generally Not Permitted . . . . . . . . . . . . . . . . . . . . . . . . 52
2) Exceptions . . . . . . . . . . . . . . . . . . . . . . . . . . . . . . . . 52
a) To Rebut Charge of Fabrication Based on Improper Motive . . 52
b) When Witness Impeached on Other Non-Character Ground . 52
3) Also Admissible as Substantive Evidence . . . . . . . . . . . . . . . 53
F. OBJECTIONS, EXCEPTIONS, OFFERS OF PROOF . . . . . . . . . . . . . . . 53
1. Objections . . . . . . . . . . . . . . . . . . . . . . . . . . . . . . . . . . . . . 53
a. Objections to Trial Testimony . . . . . . . . . . . . . . . . . . . . . . . . 53
b. Objections to Deposition Testimony . . . . . . . . . . . . . . . . . . . . . 53
c. Specificity of Objections . . . . . . . . . . . . . . . . . . . . . . . . . . . 54
1) General Objection Sustained . . . . . . . . . . . . . . . . . . . . . . 54
2) General Objection Overruled . . . . . . . . . . . . . . . . . . . . . 54
3) Specific Objection Sustained . . . . . . . . . . . . . . . . . . . . . . 54
d. “Opening the Door” . . . . . . . . . . . . . . . . . . . . . . . . . . . . . 54
e. Rule of Completeness—Writings and Recorded Statements . . . . . . . 54
f. Motion to Strike—Unresponsive Answers . . . . . . . . . . . . . . . . . 54
2. Exceptions . . . . . . . . . . . . . . . . . . . . . . . . . . . . . . . . . . . . . 54
3. Offers of Proof . . . . . . . . . . . . . . . . . . . . . . . . . . . . . . . . . . . 55
a. Witness Offer . . . . . . . . . . . . . . . . . . . . . . . . . . . . . . . . . 55
b. Lawyer Offer . . . . . . . . . . . . . . . . . . . . . . . . . . . . . . . . . 55
c. Tangible Offer . . . . . . . . . . . . . . . . . . . . . . . . . . . . . . . . 55
G. TESTIMONIAL PRIVILEGES . . . . . . . . . . . . . . . . . . . . . . . . . . . . 55
1. Federal Rules—No Specific Privilege Provisions . . . . . . . . . . . . . . . . 55
2. General Considerations . . . . . . . . . . . . . . . . . . . . . . . . . . . . . . 55
a. Persons Who May Assert a Privilege . . . . . . . . . . . . . . . . . . . . 56
b. Confidentiality . . . . . . . . . . . . . . . . . . . . . . . . . . . . . . . . 56
c. Comment on Privilege Forbidden . . . . . . . . . . . . . . . . . . . . . . 56
d. Waiver . . . . . . . . . . . . . . . . . . . . . . . . . . . . . . . . . . . . . 56
e. Eavesdroppers . . . . . . . . . . . . . . . . . . . . . . . . . . . . . . . . 56
3. Attorney-Client Privilege . . . . . . . . . . . . . . . . . . . . . . . . . . . . . 56
a. Attorney-Client Relationship . . . . . . . . . . . . . . . . . . . . . . . . 57
1) Client . . . . . . . . . . . . . . . . . . . . . . . . . . . . . . . . . . . 57
2) Representative of Client . . . . . . . . . . . . . . . . . . . . . . . . 57
3) Attorney . . . . . . . . . . . . . . . . . . . . . . . . . . . . . . . . . 57
4) Representative of Attorney . . . . . . . . . . . . . . . . . . . . . . . 57
5) Corporation as Client . . . . . . . . . . . . . . . . . . . . . . . . . . 57
b. Confidential Communication . . . . . . . . . . . . . . . . . . . . . . . . 57
1) Communications Through Agents . . . . . . . . . . . . . . . . . . . 57
a) Examination by Doctor . . . . . . . . . . . . . . . . . . . . . . 58
2) No Privilege Where Attorney Acts for Both Parties . . . . . . . . . 58
c. Client as Holder of Privilege . . . . . . . . . . . . . . . . . . . . . . . . . 58
EVIDENCE ix.
d. Duration of Privilege . . . . . . . . . . . . . . . . . . . . . . . . . . . . . 58
e. Nonapplicability of the Privilege . . . . . . . . . . . . . . . . . . . . . . 58
1) Legal Advice in Aid of Future Wrongdoing . . . . . . . . . . . . . . 58
2) Claimants Through Same Deceased Client . . . . . . . . . . . . . . 59
3) Dispute Between Attorney and Client . . . . . . . . . . . . . . . . . 59
f. Waiver of the Privilege . . . . . . . . . . . . . . . . . . . . . . . . . . . . 59
g. Attorney’s Work Product . . . . . . . . . . . . . . . . . . . . . . . . . . 59
h. Limitations on Waiver of Attorney-Client Privilege and Work Product
Rule . . . . . . . . . . . . . . . . . . . . . . . . . . . . . . . . . . . . . . 59
1) Disclosure in Federal Proceeding . . . . . . . . . . . . . . . . . . . 59
2) Disclosure in State Proceeding . . . . . . . . . . . . . . . . . . . . . 59
4. Physician-Patient Privilege . . . . . . . . . . . . . . . . . . . . . . . . . . . . 59
a. Elements of Physician-Patient Privilege . . . . . . . . . . . . . . . . . . . 60
1) Professional Member of Relationship Must Be Present . . . . . . . 60
2) Information Must Be Acquired While Attending Patient . . . . . . 60
3) Information Must Be Necessary for Treatment . . . . . . . . . . . . 60
b. Nonapplicability of the Privilege . . . . . . . . . . . . . . . . . . . . . . 60
1) Patient Puts Physical Condition in Issue . . . . . . . . . . . . . . . . 60
2) In Aid of Wrongdoing . . . . . . . . . . . . . . . . . . . . . . . . . . 60
3) Dispute Between Physician and Patient . . . . . . . . . . . . . . . . 60
4) Agreement to Waive the Privilege . . . . . . . . . . . . . . . . . . . 61
5) Federal Cases Applying Federal Law of Privilege . . . . . . . . . . 61
c. Criminal Proceedings . . . . . . . . . . . . . . . . . . . . . . . . . . . . 61
d. Patient Holds the Privilege . . . . . . . . . . . . . . . . . . . . . . . . . . 61
5. Psychotherapist/Social Worker-Client Privilege . . . . . . . . . . . . . . . . 61
6. Privileges Related to Marriage . . . . . . . . . . . . . . . . . . . . . . . . . . 61
a. Spousal Immunity—Privilege Not to Testify in Criminal Cases . . . . . 61
1) Federal Courts—Privilege Belongs to Witness-Spouse . . . . . . . . 62
2) Some State Courts—Privilege Belongs to Party-Spouse . . . . . . . 62
3) Valid Marriage Required . . . . . . . . . . . . . . . . . . . . . . . . 62
4) Immunity May Be Asserted Only During Marriage . . . . . . . . . 62
b. Privilege for Confidential Marital Communications . . . . . . . . . . . . 62
1) Both Spouses Hold Privilege . . . . . . . . . . . . . . . . . . . . . . 62
2) Elements of the Privilege . . . . . . . . . . . . . . . . . . . . . . . . 62
a) Marital Relationship . . . . . . . . . . . . . . . . . . . . . . . . 62
b) Reliance upon Intimacy . . . . . . . . . . . . . . . . . . . . . . 62
c. Nonapplicability of Privileges . . . . . . . . . . . . . . . . . . . . . . . . 63
7. Privilege Against Self-Incrimination . . . . . . . . . . . . . . . . . . . . . . . 63
a. “Incriminating” Defined . . . . . . . . . . . . . . . . . . . . . . . . . . . 63
b. When Privilege Applies . . . . . . . . . . . . . . . . . . . . . . . . . . . 63
8. Clergy-Penitent Privilege . . . . . . . . . . . . . . . . . . . . . . . . . . . . . 63
9. Accountant-Client Privilege . . . . . . . . . . . . . . . . . . . . . . . . . . . 63
10. Professional Journalist Privilege . . . . . . . . . . . . . . . . . . . . . . . . . 63
11. Governmental Privileges . . . . . . . . . . . . . . . . . . . . . . . . . . . . . 64
a. Identity of Informer . . . . . . . . . . . . . . . . . . . . . . . . . . . . . 64
1) Privilege Claimed by Government . . . . . . . . . . . . . . . . . . . 64
2) No Privilege If Identity Otherwise Voluntarily Disclosed . . . . . . 64
3) Judge May Dismiss If Informer’s Testimony Crucial . . . . . . . . . 64
x. EVIDENCE
b. Official Information . . . . . . . . . . . . . . . . . . . . . . . . . . . . . 64
H. EXCLUSION AND SEQUESTRATION OF WITNESSES . . . . . . . . . . . . . 64
I. WITNESSES EXAMINED OR CALLED BY THE COURT . . . . . . . . . . . . 64
2. Former Testimony . . . . . . . . . . . . . . . . . . . . . . . . . . . . . . . . . 73
a. Identity of Parties . . . . . . . . . . . . . . . . . . . . . . . . . . . . . . 74
b. Identity of Subject Matter . . . . . . . . . . . . . . . . . . . . . . . . . . 74
c. Opportunity to Develop Testimony at Prior Hearing . . . . . . . . . . . 74
d. Under Oath . . . . . . . . . . . . . . . . . . . . . . . . . . . . . . . . . . 74
e. Use in Criminal Proceedings . . . . . . . . . . . . . . . . . . . . . . . . 74
3. Statements Against Interest . . . . . . . . . . . . . . . . . . . . . . . . . . . . 75
a. Requirements of the Statement . . . . . . . . . . . . . . . . . . . . . . . 75
b. Risk of Civil Liability . . . . . . . . . . . . . . . . . . . . . . . . . . . . 75
c. Risk of Criminal Liability . . . . . . . . . . . . . . . . . . . . . . . . . . 75
1) Third-Party Confession Allowed . . . . . . . . . . . . . . . . . . . . 75
2) Co-Defendant’s Confession May Not Be Admissible . . . . . . . . . 76
d. “Statement” Means Single Remark . . . . . . . . . . . . . . . . . . . . . 76
4. Dying Declarations—Statements Under Belief of Impending Death . . . . . 76
5. Statements of Personal or Family History . . . . . . . . . . . . . . . . . . . . 76
a. Statement Need Not Have Been Made Before Controversy . . . . . . . . 76
b. Usually Declarant Must Be a Family Member . . . . . . . . . . . . . . . 77
c. Personal Knowledge Required . . . . . . . . . . . . . . . . . . . . . . . . 77
d. Other Ways to Prove Pedigree . . . . . . . . . . . . . . . . . . . . . . . . 77
6. Statements Offered Against Party Procuring Declarant’s Unavailability . . . 77
D. HEARSAY EXCEPTIONS—DECLARANT’S AVAILABILITY
IMMATERIAL . . . . . . . . . . . . . . . . . . . . . . . . . . . . . . . . . . . . . 77
1. Present State of Mind . . . . . . . . . . . . . . . . . . . . . . . . . . . . . . . 77
a. Rationale . . . . . . . . . . . . . . . . . . . . . . . . . . . . . . . . . . . 77
b. When Admissible . . . . . . . . . . . . . . . . . . . . . . . . . . . . . . . 78
1) State of Mind Directly in Issue and Material to the Controversy . . 78
2) Offered to Show Subsequent Acts of Declarant . . . . . . . . . . . . 78
c. Statements of Memory or Belief Generally Not Admissible . . . . . . . . 78
2. Excited Utterances . . . . . . . . . . . . . . . . . . . . . . . . . . . . . . . . . 78
a. Startling Event Required . . . . . . . . . . . . . . . . . . . . . . . . . . 78
b. Declaration Must Be Made While Under Stress of Excitement . . . . . . 78
3. Present Sense Impressions . . . . . . . . . . . . . . . . . . . . . . . . . . . . 79
a. Comment Made Concurrently with Sense Impression . . . . . . . . . . . 79
b. Safeguards . . . . . . . . . . . . . . . . . . . . . . . . . . . . . . . . . . 79
4. Declarations of Physical Condition . . . . . . . . . . . . . . . . . . . . . . . . 79
a. Present Bodily Condition—Admissible . . . . . . . . . . . . . . . . . . . 79
b. Past Bodily Condition—Admissible If to Assist Diagnosis or
Treatment . . . . . . . . . . . . . . . . . . . . . . . . . . . . . . . . . . . 79
5. Records of a Regularly Conducted Activity—Business Records . . . . . . . . 80
a. Rationale . . . . . . . . . . . . . . . . . . . . . . . . . . . . . . . . . . . 80
b. Elements of Business Records Exception . . . . . . . . . . . . . . . . . . 80
1) “Business” . . . . . . . . . . . . . . . . . . . . . . . . . . . . . . . . 80
2) Entry Made in Regular Course of Business . . . . . . . . . . . . . . 80
a) Business Activity . . . . . . . . . . . . . . . . . . . . . . . . . . 80
(1) Hospital Records . . . . . . . . . . . . . . . . . . . . . . . 80
(2) Police Reports . . . . . . . . . . . . . . . . . . . . . . . . . 81
(3) The Rule of Palmer v. Hoffman—Records Prepared for
Litigation . . . . . . . . . . . . . . . . . . . . . . . . . . . 81
xii. EVIDENCE
EVIDENCE
I. GENERAL CONSIDERATIONS
E. EVIDENCE CLASSIFICATIONS
1. Direct or Circumstantial
a. Testimonial evidence is oral evidence given under oath. The witness responds to the
questions of the attorneys.
F. LIMITED ADMISSIBILITY
II. RELEVANCE
A. INTRODUCTION
Relevance, in the sense of probativeness, has to do with the tendency of evidence to prove or
disprove a material issue, to render it more probably true, or untrue, than it would have been
4. EVIDENCE
without the particular evidence. Relevance is concerned with the substance or content of the
evidence, not with the form or manner in which it is offered (e.g., hearsay rule, best evidence
rule). It can be stated, as a general proposition, that all relevant evidence is admissible if it is
offered in an unobjectionable form and manner. (As usual, there are some exceptions to this
generalization, since some perfectly relevant evidence that is in the proper form is excluded for
policy reasons.)
B. DETERMINING RELEVANCE
Relevant evidence is evidence having any tendency to make the existence of any fact that is of
consequence to the determination of an action more or less probable than it would be without the
evidence. [Fed. R. Evid. 401] Note that this definition of relevance includes materiality, since it
requires that the disputed fact be of consequence to the determination of the action. The basic
questions to ask regarding relevancy are: “What proposition is the evidence being used to prove?
Is this a material issue in the case? Is the evidence probative of that proposition?” This type of
relevance is sometimes called “logical relevance.”
inference about the subject occurrence, the quality of the inference depends on the similarity
between the other happening and the one in issue. The following are examples of relevant
similar occurrences.
a. Causation
Complicated issues of causation may often be established by evidence that concerns
other times, events, or persons. For example, evidence that other homes in the same area
were damaged by defendant’s blasting operations is some evidence that the damage to
plaintiff’s home was caused by defendant’s activities.
But if evidence is introduced that the party has made previous similar false claims, such
evidence is usually relevant, under a common scheme or plan theory, to prove that the
present claim is likely to be false. Likewise, where the prior claim was for an injury to
the same portion of plaintiff’s body that she claims was injured in the present case,
evidence of the prior claim or injury may be relevant to prove that her present claim is
false or exaggerated.
Example: In an action against a school board for excluding a black child from
school, similar exclusions of black children will be admissible into
evidence to show intent.
e. Rebutting Claim of Impossibility
The requirement that prior occurrences be similar to the litigated act may be relaxed
when used to rebut a claim or defense of impossibility. For example, if defendant denies
negligent speeding on the ground that his automobile could never go above 50 m.p.h.,
plaintiff may rebut by showing that on other occasions, even under different circum-
stances, the vehicle was traveling at 75 m.p.h.
f. Sales of Similar Property
Evidence of sales of similar personal or real property that are not too remote in time is
admissible to prove value. However, unlike commonly sold items of personal property,
each parcel of real property is considered unique. Thus, the problem of producing
evidence of other transactions requires a preliminary finding that the character, usage,
proximity, date of sale, etc., are sufficiently similar to the property in issue. Evidence
of prices quoted in mere offers is not admissible because to determine the sincerity of
these offers would lead to collateral disputes. However, offers by a party to the present
action to buy or sell the property may be admissible as admissions.
g. Habit
Habit describes one’s regular response to a specific set of circumstances (e.g., “she
always takes a staircase two steps at a time”). In contrast, character describes one’s
disposition in respect to general traits (e.g., “she’s always in a hurry”). Since habits are
more specific and particularized, evidence of habit is relevant and can be introduced
in circumstances when it is not permissible to introduce evidence of character. Thus,
under Federal Rule 406, evidence of a person’s habit may be admitted to prove that on a
particular occasion the person acted in accordance with the habit.
Example: Evidence could be introduced to show that a driver habitually failed to
stop at a certain stop sign as circumstantial evidence that she failed to
stop at the time in question. In contrast, evidence cannot be introduced
to show that a person is a careless driver since that is closer to character
than habit.
Many states either do not admit evidence of habit to show that a particular act occurred,
or else limit admissibility to those cases where there are no eyewitnesses. Federal
Rule 406 admits habit evidence freely and abandons the “no eyewitness” requirement.
Even where admissible, however, the habit must be shown to be a regular response to a
repeated specific situation.
Certain items of evidence may be directed to a material issue in the case and may be very
probative of that issue, but they are excluded because of predictable policies designed to ensure
an orderly and efficient proceeding and to encourage certain public policy solutions to legal
problems. For example, inflammatory matter, which may be very probative of the issues, will not
be admitted because of the potential prejudicial effect on the jury.
1. Liability Insurance
b. When Admissible
Proof that a person carried liability insurance may be admissible and relevant for other
8. EVIDENCE
purposes. Issue identification is important in these cases, since proof of the fact that the
defendant maintained insurance may be used:
3) As Part of Admission
An admission of liability may be so coupled with a reference to insurance coverage
that the reference to insurance cannot be severed without lessening its value as an
admission of liability.
Example: “Don’t worry; my insurance company will pay off.”
b. When Admissible
Although evidence of subsequent repairs is not admissible to prove negligence, etc., this
evidence may still be admissible for other purposes. Some of these purposes are:
The Federal Rules also exclude conduct or statements made in the course of negotiating a
compromise, as well as the offer to compromise itself; therefore, admissions of fact made
during compromise negotiations are inadmissible. This position encourages settlements by
allowing complete candor between the parties in negotiations. However, conduct or state-
ments made during compromise negotiations regarding a civil dispute with a governmental
regulatory, investigative, or enforcement authority are not excluded when offered in a
criminal case. [Fed. R. Evid. 408] Note that Rule 408 does not protect preexisting informa-
tion simply because it is presented to one’s opponent during compromise negotiations; e.g.,
one may not immunize otherwise admissible information under the guise of disclosing it
during compromise negotiations.
a. Must Be a Claim
Although the filing of a suit is not a prerequisite for this exclusionary rule, there must be
some indication, express or implied, that a party is going to make some kind of claim.
Thus, a party’s volunteered admission of fact accompanying an offer to settle immedi-
ately following the incident is usually admissible because there has not been time for the
other party to indicate an intent to make a claim.
withdrawn plea of guilty as an admission is deemed offset by the prejudicial effect of the
evidence. Moreover, it is felt that the judge who permitted the withdrawal of the guilty plea
must have decided that there was a good reason to withdraw it and, under these circum-
stances, the significance of the initial plea is minimal. Most courts exclude offers to plead
guilty on reasoning similar to that advanced for not admitting offers to compromise as proof
of liability in civil cases.
a. Waiver
The protection of Rule 410 for plea negotiations may be validly waived unless there is
an affirmative indication that the defendant entered the waiver agreement unknowingly
or involuntarily. [United States v. Mezzanatto, 513 U.S. 196 (1995)]
Example: Defendant, who was charged with a drug offense, wished to arrange
a deal with the government in exchange for his cooperation. As a
prerequisite to this discussion, Prosecutor required that Defendant (i)
be completely truthful and (ii) agree that any statements made by him
in the course of the plea negotiations could be used to impeach him if
he testified in a contrary fashion at trial. Defendant agreed. If at some
point thereafter the discussion breaks off and Defendant is tried on the
charges, Prosecutor may use statements made in the plea negotiations to
impeach Defendant. [United States v. Mezzanatto, supra]
character is permitted only in a few instances, such as where character is itself one of
the ultimate issues in the case. Note, however, that specific acts may be admissible if
relevant for some other purpose (see 6.a., infra).
b. Opinion Testimony
Witnesses who know the person may testify regarding their opinions about the person’s
character.
c. Testimony as to Person’s General Reputation in Community
Testimony by witnesses as to a person’s general reputation in the community is in some
sense hearsay, since reputation is really what people say about a person. On the other
hand, because reputation is a general indication of character, and because it involves
fewer side issues than either of the above methods, it is the most common means of
showing character.
3. Generally Not Admissible in Civil Cases
Evidence of character to prove the conduct of a person in the litigated event is generally not
admissible in a civil case. [Fed. R. Evid. 404(a)] The reasons given are that the slight proba-
tive value of character is outweighed by the danger of prejudice, the possible distraction of
the jury from the main question in issue, and the possible waste of time required by exami-
nation of collateral issues.
Examples: 1) Defendant may not introduce evidence that she is generally a cautious
driver to prove that she was not negligent on the day in question.
2) Plaintiff may not introduce evidence that the defendant is usually a reckless
driver to prove that she was negligent on the day in question.
Such circumstantial use of prior behavior patterns for the purpose of drawing the inference
that, at the time and place in question, the actor probably acted in accord with her prior
behavior pattern is not permitted. A person’s general behavior patterns (as distinguished
from her habits and business routines) are irrelevant and inadmissible in evidence.
a. Exception—When Character Is Directly in Issue
When proof of a person’s character, as a matter of substantive law, is an essential
element of a claim or defense in a civil action, it is said that character is “directly in
issue.” Although character is rarely an essential issue in a civil case, character evidence
is admissible in such circumstances.
Examples: 1) In a defamation action, when D is being sued for calling P a thief and
pleads as an affirmative defense that she spoke the truth (i.e., that P is
indeed a thief), P’s character is clearly in issue.
2) When an employer is charged with negligently retaining an employee
“of unstable and violent disposition,” the character of the employee is
also in issue.
Compare: In a civil action for damages based on assault or battery, the defendant’s
claim that he acted in self-defense does not put either the plaintiff-
victim’s or the defendant’s character for violence or peacefulness into
issue. On the issue of who struck first, the substantive law does not
require proof of either party’s character.
12. EVIDENCE
When character is directly in issue, almost all courts will admit evidence of specific
acts that show this character (e.g., in Example 1) above, D may offer evidence that on
different occasions P has stolen things to show that he is a thief). Under the Federal
Rules, any of the types of evidence (reputation, opinion, or specific acts) may be used
to prove character when character is directly in issue. [Fed. R. Evid. 405(b)]
The rationale is that even though the evidence is of some relevance, the prosecution should
not be permitted to show that the defendant is a bad person, since the jury might then
decide to convict her regardless of her guilt of the crime charged. However, since the life or
liberty of the defendant is at stake, she should be allowed to introduce evidence of her good
character since it may have a tendency to show that she did not commit the crime charged.
1) Cross-Examination
The prosecution may test the character witness by cross-examination regarding
the basis for his opinion or knowledge of the reputation that he has testified about.
In most jurisdictions, one is allowed to inquire on cross-examination whether the
reputation witness has heard of particular instances of the defendant’s miscon-
duct pertinent to the trait in question. The rationale is that since the reputation
witness relates what he has heard, the inquiry tests the accuracy of his hearing
and reporting. Since the character witness may now testify in the form of opinion
as well as by reputation, it follows that the basis of the opinion can be exposed.
Thus, under Federal Rule 405(a), cross-examination inquiry is allowable as to
EVIDENCE 13.
whether the opinion witness knows of, as well as whether he has heard of, specific
instances of misconduct. The distinction in form between “Have you heard” and
“Do you know” is eliminated by the statement in Rule 405(a), which provides that
“on cross-examination of the character witness, the court may allow an inquiry
into relevant specific instances of the person’s conduct.” Note that if the witness
denies knowledge of these specific instances of conduct, the prosecutor may not
prove them by extrinsic evidence; he is limited to inquiry on cross-examination.
a. Defendant’s Initiative
The defendant may introduce reputation or opinion evidence of a bad character trait
of the alleged crime victim when it is relevant to show the defendant’s innocence.
However, by specific exception, this rule does not extend to showing the bad character
of rape victims.
Example: In an assault or murder prosecution where the defendant claims self-
defense, she may introduce evidence of the victim’s violent character as
tending to show that the victim was the aggressor.
b. Prosecution Rebuttal
Once the defendant has introduced evidence of the alleged victim's bad character for a
pertinent trait, the prosecution may counter with reputation or opinion evidence of (i)
the victim’s good character for the same trait, or (ii) the defendant’s bad character for
the same trait. [Fed. R. Evid. 404(a)]
Example: Defendant is charged with the murder of Victim. Defendant pleads self-
defense and offers evidence that Victim was a violent person. Prosecutor
can rebut such evidence with evidence that Victim was a nonviolent
person and/or with evidence that Defendant is a violent person.
3) Procedure
To offer evidence under the above exceptions, the party must file a motion 14 days
before trial describing the evidence and its purpose, and must serve the motion
on all parties and notify the victim. Before admitting the evidence, the court must
conduct an in camera hearing and afford the victim and the parties a right to be
heard. [Fed. R. Evid. 412(c)]
a) Motive
The commission of a prior crime may be evidence of a motive to commit the
crime for which the defendant is accused.
b) Intent
In many crimes, such as forgery, passing counterfeit money, larceny by trick,
and receiving stolen property, intent is the gravamen of the crime. Evidence
that defendant committed prior, similar wrongful acts is admissible to estab-
lish guilty knowledge and to negate good faith.
d) Identity
Evidence, including misconduct, that connects this defendant to the crime
(e.g., theft of gun used in later crime) is admissible. Similarly, evidence that
the accused committed prior criminal acts that are so distinctive as to operate
as a “signature” may be introduced to prove that the accused committed the
act in question (modus operandi).
f) Other
Similar acts or related misconduct may be used to prove opportunity, knowl-
edge, or any relevant fact other than the accused’s general bad character or
criminal disposition.
sexual assault or child molestation. The party who intends to offer this evidence must
disclose the evidence to the defendant 15 days before trial (or later with good cause).
[Fed. R. Evid. 413 - 415]
1) The streets in Manhattan are numbered east and west from Fifth
Avenue and that the odd numbers are on the north side of the street.
2) Many people are subject to low blood pressure and poor circulation.
2) The court will accept without proof that February 14, 1999, was a
Sunday by reference to a calendar.
a. Requirement of a Request
In instances where the court does not take judicial notice of a fact on its own accord,
the general rule is that a party must formally request (e.g., through pleadings or an oral
motion) that notice be taken.
on the other hand, the jury is instructed that it may, but is not required to, accept as
conclusive any fact judicially noticed.
a. Federal public law—the United States Constitution, federal treaties, public acts of
Congress, and federal case law.
b. State public law—the constitution, public statutes, and common law of the states.
c. Official regulations—the official compilation of codes and rules and regulations of the
forum state and the federal government, except those relating to internal organization or
management of a state agency.
A. IN GENERAL
EVIDENCE 19.
2. Special Problems
This form of proof, which allows the triers of fact to reach conclusions based upon their own
perceptions rather than relying upon those of witnesses, frequently involves special problems.
Often there is concern regarding proper authentication of the “object.” Additionally, the
possibility exists that physical production of the thing may be too burdensome or may
inspire prejudicial emotions outweighing its probative value to the litigation.
B. TYPES OF REAL EVIDENCE
1. Direct
Real evidence may be direct evidence; i.e., it may be offered to prove the facts about the
object as an end in itself. For example, in a personal injury case, evidence of a permanent
injury could be introduced by an exhibition of the injury itself to the trier of fact.
2. Circumstantial
Real evidence may also be circumstantial; i.e., facts about the object are proved as a basis
for an inference that other facts are true. For example, in a paternity case, the trier of fact
may be shown the child for the purpose of showing that she is of the same race as the alleged
father. In this case, the trier of fact is being asked to draw an inference that, since the child
and alleged father are of the same race, the paternal relationship exists.
3. Original
Real evidence may be original; i.e., it may have had some connection with the transaction
that is in question at the trial. An example of this kind of evidence would be an alleged
murder weapon.
4. Prepared
Real evidence may also be prepared; e.g., sketches or models may be made to be shown to
the trier of fact. This category of real evidence is called “demonstrative evidence.”
1. Authentication
The object must first be identified as being what the proponent claims it to be. Real evidence
is commonly authenticated by recognition testimony or by establishing a chain of custody.
a. Recognition Testimony
If the object has significant features that make it identifiable upon inspection, a witness
may authenticate the object by testifying that the object is what the proponent claims it is.
20. EVIDENCE
Examples: 1) If a prosecutor offers a knife into evidence and claims that the knife
is the very weapon used in the murder, the object may be authenticated
by a witness who testifies that he can identify the knife as the one found
next to the deceased.
b. Chain of Custody
If the evidence is of a type that is likely to be confused or can be easily tampered with,
the proponent of the object must present evidence of chain of custody. The proponent
of the evidence must show that the object has been held in a substantially unbroken
chain of possession. The proponent need not negate all possibilities of substitution or
tampering, but must show adherence to some system of identification and custody.
Example: A custodial chain—from the taking, to the testing, to the exhibiting of
the sample—must be established before evidence of a blood alcohol test
will be admitted.
3. Legal Relevance
Assuming the object has been properly identified and is probative, the discretion of the trial
judge is called upon to decide whether some auxiliary policy or principle outweighs the need
to admit the real evidence. Such policies limiting the use of real evidence frequently concern:
b. Indecency or impropriety; or
c. Undue prejudice where the probative value of the object or exhibit is outweighed by the
danger of unfair prejudice.
and are not given to the jury during its deliberations. These items are not represented to be
reproductions of the real thing, but are merely used as aids to testimony.
Example: A doctor may use a model of an average male skeleton to explain his testi-
mony. The skeleton may be marked for identification in order to preserve the
record, but it is not admitted into evidence.
Compare: If the skeleton was offered as a reproduction of the bone structure of the
deceased, assuming there is no undue prejudice, it may be admitted into
evidence on a showing that it accurately represents the bone structure of the
deceased.
4. Exhibition of Injuries
The exhibition of injuries in a personal injury or criminal case is generally permitted, but the
court has discretion to exclude this evidence if the exhibition would result in unfair prejudice.
6. Demonstrations
The court, in its discretion, may permit experiments or demonstrations to be performed in
the courtroom.
c. Scientific Experiments
The judge may permit scientific experiments to be performed in the courtroom
provided:
1) The conditions are substantially similar to those that attended the original event,
and
2) The experiment will not result in undue waste of time or confusion of the issues.
V. DOCUMENTARY EVIDENCE
A. IN GENERAL
Documentary evidence, like other kinds of evidence, must be relevant in order to be admissible.
In the case of writings, the authenticity of the document is one aspect of its relevancy. Of course,
documentary evidence, even if fully authenticated and relevant, may be excluded if it violates a
rule of competency such as the best evidence or hearsay rule. Whenever any problem or question
concerns a document, you should consider three separate and distinct possible barriers to admis-
sibility (authentication, best evidence, and hearsay).
B. AUTHENTICATION
Before a writing or any secondary evidence of its content may be received in evidence, the writing
must be authenticated by proof showing that the writing is what the proponent claims it is. The
writing is usually not self-authenticating. It needs a testimonial sponsor or shepherding angel to
prove that the writing was made, signed, or adopted by the particular relevant person. [Fed. R.
Evid. 901 - 903]
3. Evidence of Authenticity
In general, a writing may be authenticated by any evidence that serves to establish its
EVIDENCE 23.
authenticity. The Federal Rules do not limit the methods of authentication, but rather list
several examples of proper authentication. [Fed. R. Evid. 901]
a. Admissions
A writing may be authenticated by evidence that the party against whom the writing is
offered has either admitted its authenticity or acted upon the writing as authentic.
b. Testimony of Eyewitness
A writing may be authenticated by testimony of one who sees it executed or hears it
acknowledged. Modern statutes eliminate the common law necessity of producing a
subscribing witness, unless specifically required by statute. [Fed. R. Evid. 903] If testi-
mony of a subscribing witness is required (e.g., in authenticating a will), his denial or
failure to recollect the execution of the writing does not preclude authentication by other
evidence.
c. Handwriting Verifications
A writing may also be authenticated by evidence of the genuineness of the handwriting
of the maker.
1) Nonexpert Opinion
A lay witness who has personal knowledge of the handwriting of the supposed
writer may state his opinion as to whether the document is in that person’s
handwriting. (This is an exception to the opinion rule, see VI.C.1., infra.) Note,
however, that a nonexpert cannot become familiar with the handwriting merely for
the purpose of testifying.
2) Comparison of Writings
An expert witness or the trier of fact (e.g., jury) can determine the genuineness of
a writing by comparing the questioned writing with another writing proved to be
genuine. (Note that authentication by comparison is not limited to handwriting.
Fingerprints, blood, hair, clothing fibers, and numerous other things can be authen-
ticated by comparison with authenticated specimens.)
d. Ancient Documents
Under the Federal Rules, a document may be authenticated by evidence that it:
(ii) Is in such condition as to be free from suspicion concerning its authenticity; and
(iii) Was found in a place where such writing, if authentic, would likely be kept.
g. Photographs
As a general rule, photographs are admissible only if identified by a witness as a
portrayal of certain facts relevant to the issue and verified by the witness as a correct
representation of those facts. It suffices if the witness who identifies the photograph is
familiar with the scene or object that is depicted. In general, it is not necessary to call
the photographer to authenticate the photograph.
a. When Necessary
Not all oral statements need to be authenticated; only where the identity of the speaker
is important (e.g., admission by a party) is authentication required.
EVIDENCE 25.
b. Methods of Authentication
1) Voice Identification
A voice, whether heard firsthand or through a device (e.g., a tape recording) may
be identified by the opinion of anyone who has heard the voice at any time. Thus,
in contrast to the rule for handwriting verification, a person can become familiar
with a voice after litigation has begun and for the sole purpose of testifying.
2) Telephone Conversations
Statements made during a telephone conversation may be authenticated by one of
the parties to the call who testifies to one of the following:
b) The speaker has knowledge of certain facts that only a particular person
would have.
c) He called, for example, Mr. A’s telephone number, and a voice answered,
“This is Mr. A” or “This is the A residence.” This authenticates the conversa-
tion as being with Mr. A or his agent.
d) He called the person’s business establishment and talked with the person
answering the phone about matters relevant to the business. This is sufficient
to show that the person answering the phone held a position in the business.
5. Self-Authenticating Documents
Contrary to the general rule, which requires testimonial sponsorship, there are certain
writings that are said to “prove themselves” or to be “self-identifying” on their face. Federal
Rule 902 specifically provides that extrinsic evidence of authenticity is not required as to the
following:
b. Foreign public documents signed by a person authorized by the laws of that country;
c. Certified copies of public records (i.e., records, reports, and recorded documents);
f. Trade inscriptions, signs, tags, or labels purporting to have been affixed in the course
of business and indicating ownership, control, or origin;
h. Commercial paper, signatures thereon, and documents relating thereto, to the extent
provided by general commercial law; and
2) Witness who memorized mileage recorded on car sticker for a certain date,
and who had no other source of knowledge on this significant litigated issue,
may not testify as to the mileage without establishing a reason for the unavail-
ability of the writing.
Examples: 1) Witness may testify orally that he paid for goods received without
producing the receipt that was given.
2) Facts such as birth, marriage, age, and death may be proved orally,
although certificates evidencing these facts are in existence. However,
since a divorce is effective only by a judicial decree, the best evidence
rule requires that the fact of divorce be proved by the decree itself.
The above examples are in contrast to those writings that are considered as essential
repositories of the facts recorded. Written contracts, deeds, wills, and judgments are
viewed as such repositories—they are considered written transactions—and as such are
within the rule.
d. Public Records
The best evidence rule is modified so that a proponent may offer into evidence a copy of
an official record or a copy of a document that has been recorded and filed. Such a copy
must be certified as correct by the custodian of the document or other person autho-
rized to do so, or testified to be correct by a person who compared it to the original.
[Fed. R. Evid. 1005] The purpose of this exception is to prevent the loss or absence of
public documents due to litigation.
28. EVIDENCE
b. “Original”
An original is the writing or recording itself or any counterpart intended by the person
executing it to have the same effect as an original. With respect to a photograph,
“original” includes the negative or any print from it. For computer-stored data, the
original is any printout or other output readable by sight.
Example: D types a letter to P that is defamatory of P. D sends the letter to P
himself and a photocopy of the letter to the newspaper. In P’s defama-
tion action, the document legally operative to create tort liability is the
“published” photocopy, not the letter sent to P.
c. Admissibility of Duplicates
The Federal Rules define a duplicate as “a counterpart produced by a mechanical,
photographic, chemical, electronic, or other equivalent process or technique that
accurately reproduces the original.” [Fed. R. Evid. 1001(e)] A duplicate is thus an exact
copy of an original (e.g., a carbon copy or photocopy) made by mechanical means.
Duplicates are admissible to the same extent as originals in federal courts, unless (i)
a genuine question is raised about the original’s authenticity, or (ii) under the circum-
stances, it would be unfair to admit the duplicate in place of the original. [Fed. R. Evid.
1003] The rationale for admitting duplicates under such a relaxed standard is that by
definition these documents are exact copies of the original, and therefore their intro-
duction into evidence would be objectionable only if some question existed as to the
genuineness of the original.
5. Admissibility of Secondary Evidence of Contents
If the proponent cannot produce the original writing or recording in court, he may offer
secondary evidence of its contents in the form of copies (e.g., handwritten copies, which
would not be considered duplicates because they are not exact copies), notes, or oral testi-
mony about the contents of the original if a satisfactory explanation is given for the nonpro-
duction of the original.
a. Satisfactory Foundation
A valid excuse justifying the admissibility of secondary evidence would include:
b. Reformation of Contract
Where a party alleges facts, such as mistake, entitling him to reformation of the written
agreement, the parol evidence rule is inapplicable. This is so because the party is
asserting that, despite the apparently unambiguous contract, its terms do not in fact
constitute the agreement intended.
2) Lack of consideration;
4) Material alteration;
allegedly made at or before the time of the written contract would be barred by the
rule.
A. COMPETENCY OF WITNESSES
Witnesses are not “authenticated” in the same sense as real or documentary evidence. However,
they too must pass tests of basic reliability to establish their competence to give testimony. Unlike
the authentication situation pertaining to real or documentary proof, witnesses are generally
presumed to be competent until the contrary is demonstrated.
A diminution of any of these capacities usually goes only to the weight of the testimony and
serves to make the witness less persuasive. However, a witness can be so deficient in one or
more of these basic qualifications that she will be deemed incompetent to testify at all. The
problem of infancy is a good example for all aspects of the basic qualifications. A witness
may be too young at the time of the event to be able to accurately perceive what happened
or to be able to remember at the time of the trial. The witness may also be too young at the
time of the trial to effectively relate or communicate or appreciate the obligation to tell the
truth.
a. Ability to Observe—Perception
The issue of a witness’s ability to observe may arise in the following manner: W testi-
fies on direct to details of how an intersection automobile collision occurred. On cross-
examination, W admits that her attention was directed to the intersection by the sound
of the crash. The direct testimony regarding details of the accident occurring before the
collision will be stricken.
b. Ability to Remember—Memory
An example of a witness incompetent for this reason would be one who is suffering
from senility or amnesia.
c. Ability to Relate—Communication
The ability to relate concerns the ability of the witness to communicate effectively with
the trier of fact.
32. EVIDENCE
1) The witness must have personal knowledge of the matter he is to testify about.
The requirement of “personal knowledge” means that the witness must have
observed the matter and must have a present recollection of his observation. [Fed.
R. Evid. 602]
2) The witness must declare he will testify truthfully, by oath or affirmation. [Fed.
R. Evid. 603]
b. Use of Interpreter
If a witness requires an interpreter, the interpreter must be qualified and take an oath to
make a true translation. [Fed. R. Evid. 604]
b. Infancy
There is no precise age at which an infant is deemed competent or incompetent to
testify under oath. The competence of an infant depends on the capacity and intelli-
gence of the particular child. This test is an individual one, to be determined by the trial
judge upon preliminary examination.
EVIDENCE 33.
c. Insanity
An insane person, even one who has been adjudicated incompetent, may testify,
provided he understands the obligation to speak truthfully and possesses the capacity to
give a correct account of what he has perceived in reference to the issue in dispute.
d. Conviction of Crime
The common law disqualification of felons has been removed by statute in most states.
However, conviction of a crime may be shown to affect the credibility of the competent
witness.
e. Interest
The common law disqualification of parties or interested persons has been abolished in
most states. The only remaining vestiges of this disqualification are the so-called Dead
Man Acts, discussed later.
f. Judge as Witness
Federal Rule 605 provides that the presiding judge may not testify as a witness, and
that no objection need be made to preserve the point. The basis for this disqualification
is that when the judge is called as a witness, her role as a witness is inconsistent with
her role as presiding judge, which requires her to maintain impartiality.
g. Juror as Witness
Under Federal Rule 606, jurors are incompetent to testify before the jury in which they
are sitting. The rationale is that a juror-witness cannot impartially weigh his own testi-
mony and cannot be thoroughly cross-examined for fear of creating antagonism.
The Federal Rule also prevents a juror from testifying in post-verdict proceedings as
to matters or statements occurring during the course of jury deliberations, except that
a juror may testify as to whether “extraneous prejudicial information” or any “outside
influence” was brought to bear on any juror. Also, a juror may testify as to whether
there was a mistake in entering the verdict onto the verdict form; e.g., where the verdict
form contains a damage amount different from that agreed upon by the jury, or where
the form mistakenly states that a criminal defendant is guilty when the jury had agreed
that he was not guilty.
a. Rationale
The Dead Man Acts generally provide that a party or person interested in the event,
or his predecessor in interest, is incompetent to testify to a personal transaction or
communication with a deceased, when such testimony is offered against the represen-
tative or successors in interest of the deceased. The rationale of the statute is to protect
34. EVIDENCE
estates from perjured claims. The assumption is that the survivor claimant may lie,
since the deceased cannot talk back. Because death has silenced one party, the statute
closes the mouth of the living person who, being interested in the litigation’s outcome,
wishes to testify on her own behalf against someone who is suing or defending in a
representative capacity (e.g., executor, administrator, heir, legatee, devisee).
b. Common Elements
Most Dead Man Acts have the following common elements and applications:
2) Protected Parties
The statute is designed to protect those who claim directly under the decedent.
They usually include an executor, administrator, heir, legatee, and devisee. If a
protected party is on either side of the lawsuit (suing or defending), the statute
applies to prevent an interested person from testifying on his own behalf.
Examples: 1) Plaintiff sues the executor of the estate for a debt owed by the
decedent. The executor is a protected party and the act applies.
3) Interested Person
A person is “interested in the event” if he stands to gain or lose by the direct
and immediate operation of the judgment, or if the judgment may be used for or
against him in a subsequent action. Thus, in states where a spouse has an inchoate
right in the other spouse’s property, both spouses may be interested and incompe-
tent to testify. Similarly, a shareholder of a corporation and the co-maker of a note
may be disqualified under the rule.
a) Predecessor in Interest
Most Dead Man Acts disqualify not only the person interested, but also the
predecessor in interest.
Example: If A assigns to B a claim against Decedent, and B sues the
estate, both A and B are incompetent. B is interested in
the event; A is the person from, through, or under whom B
derived his interest.
b) “Door Openers”
The estate representatives and those claiming under the decedent may waive
the protection of the statute. Common provisions for waiver include:
(1) If the protected party calls the interested person to testify about the
transaction, the interested person may explain all matters about which he
is examined.
(3) Where there is a failure to make timely and proper objection. Objection
is to the incompetency of the witness, not to the incompetency of the
testimony.
1. Leading Questions
a. Generally Objectionable
A question is leading and generally objectionable on direct examination when it
suggests to the witness the fact that the examiner expects and wants to have confirmed.
Questions calling for “yes” or “no” answers and questions framed to suggest the
answer desired are usually leading.
Example: On direct examination plaintiff is asked, “Is it true or not that at the time
in question, you were driving well within the speed limit?” The question
is leading.
36. EVIDENCE
b. When Permitted
Leading questions are permitted on cross-examination. Trial judges will usually allow
leading questions on direct examination in noncrucial areas if no objection is made:
a. Misleading
A question is misleading and thus is not permitted if it is one that cannot be answered
without making an unintended admission.
Example: “Do you still beat your wife?”
b. Compound
Questions that require a single answer to more than one question are not permitted.
Example: “Did you see and hear the intruder?”
c. Argumentative
Argumentative questions, which are leading questions that reflect the examiner’s inter-
pretation of the facts, are improper.
Example: “Why were you driving so recklessly?”
d. Conclusionary
A question that calls for an opinion or conclusion that the witness is not qualified or
permitted to make is improper.
Example: “What did your friend think about that?” The witness could not know
his friend’s thoughts, and is not permitted to give his opinion as to his
friend’s thoughts.
f. Cumulative
An attorney is generally not permitted to ask a question that has already been asked
and answered. More repetition is allowed on cross-examination than on direct, but if it
is apparent that the cross-examiner is not moving forward, the judge may disallow the
question.
EVIDENCE 37.
g. Harassing or Embarrassing
The trial judge, in her discretion, may disallow cross-examination that is unduly
harassing or embarrassing.
j. Lack of Foundation
A witness must have personal knowledge as to the facts of his testimony. Insufficient
personal knowledge may subject testimony to an objection for lack of foundation.
Additionally, a party may object on the basis of lack of foundation for real evidence if
the proponent has not shown that the evidence is what he purports it to be. (See IV.C.1.,
supra.)
k. Nonresponsive Answer
A witness’s response must address only the specific question asked by the examining
attorney; otherwise the testimony is subject to being stricken for nonresponsiveness.
Example: Q. Did you leave your house on September 22?
testify fully and accurately, even after she has consulted a writing given to her on the
stand, the writing itself may be read into evidence if a proper foundation is laid for its
admissibility. This use of a memorandum as evidence of a past recollection is frequently
classified as an exception to the hearsay rule. The foundation for receipt of the writing
into evidence must include proof that:
(i) The witness at one time had personal knowledge of the facts recited in the
writing;
(ii) The writing was made by the witness or made under her direction or that it was
adopted by the witness;
(iii) The writing was timely made when the matter was fresh in the mind of the
witness;
(iv) The writing is accurate (i.e., witness must vouch for the accuracy of the writing);
and
(v) The witness has insufficient recollection to testify fully and accurately.
Remember that, under the Federal Rule, if admitted, the writing may be read into
evidence and heard by the jury, but the document itself is not received as an exhibit
unless offered by the adverse party. [Fed. R. Evid. 803(5)]
C. OPINION TESTIMONY
The word “opinion” used in this context includes all opinions, inferences, conclusions, and other
subjective statements made by a witness. A basic premise of our legal system is that, in general,
witnesses should testify as to facts within their personal knowledge and that the trier of fact
should draw any conclusions therefrom. Therefore, the general policy of the law is to restrict the
admissibility of opinion evidence, except in cases where the courts are sure that it will be neces-
sary, or at least helpful. Of course, the difference between “fact” and “opinion” is a matter of
degree. Therefore, there cannot be any clear-cut opinion rule.
where, from the nature of the subject matter, no better evidence can be obtained. In
these cases, where the event is likely to be perceived as a whole impression (e.g., intoxi-
cation, speed) rather than as more specific components, opinions by lay witnesses are
generally admitted.
b. When Admissible
In most jurisdictions and under the Federal Rules, opinion testimony by lay witnesses is
admissible when:
Some jurisdictions are stricter and allow lay opinion testimony only in cases of “neces-
sity” when it is difficult for the witness to express her perception in any form other than
opinion.
c. Procedure
Unless waived by a failure to object, a proper foundation must be laid by showing that
the witness had the opportunity to observe the event that forms the basis of her opinion.
Additionally, the court in its discretion may require a witness to state the facts observed
before stating her opinion.
2) State of Emotion
A witness would be permitted to testify that a person appeared “angry,” “was
joking,” or that two persons were “in love” or appeared to have a strong affection
for each other.
8) Intoxication
A witness who has seen a person and is able to describe that person’s actions,
words, or conduct may express an opinion as to whether that person was or was not
intoxicated. In many states, the details of the person’s appearance must be given as
a foundation for the opinion.
Example: In Defendant’s trial on a charge of driving while intoxicated,
Witness testifies for the prosecution that Defendant “smelled of
alcohol, his speech was incoherent, his eyes glassy and bloodshot,
he could not stand or walk without assistance, he was slumped over
the wheel of his vehicle,” and, finally, that he “was intoxicated.”
1) Agency or Authorization
When agency or authorization is in issue, the witness generally may not state a
conclusion as to her authorization. Rather, she must be asked by whom she was
employed and the nature, terms, and surrounding circumstances of her employ-
ment.
2) Contract or Agreement
When the existence of an express contract is in issue, a witness generally may not
state her opinion that an agreement was made. Rather, she must be asked about the
facts, the existence or nonexistence of which establish whether a contract existed.
(i) The opinion must be relevant (i.e., it must “fit” the facts of the case); and
(ii) The methodology underlying the opinion must be reliable (i.e., the proponent
of the expert testimony must satisfy the trial judge by a preponderance of
the evidence that (a) the opinion is based on sufficient facts or data; (b) the
opinion is the product of reliable principles and methods; and (c) the expert
has reliably applied the principles and methods to the facts of the case).
[Fed. R. Evid. 702; and see Kumho Tire Co. v. Carmichael, 526 U.S. 137 (1999);
Daubert v. Merrell Dow Pharmaceuticals, Inc., 509 U.S. 579 (1993)]
a) Personal Observation
If the expert has examined the person or thing about which he is testifying, he
may relate those facts observed by him and upon which he bases his opinion.
[Fed. R. Evid. 703]
Example: An expert may testify that he examined plaintiff’s leg
following the accident, and in his opinion the plaintiff
sustained a compound fracture.
42. EVIDENCE
The Federal Rules have expanded the admissibility of learned texts and treatises beyond
impeachment of experts. Statements from an established treatise may be read into the
record as substantive evidence, and may even be introduced on direct examination of
a party’s own expert. [Fed. R. Evid. 803(18)—exception to hearsay rule] There are
two important limitations: (i) an expert must be on the stand when a statement from a
treatise is read into evidence; and (ii) the relevant portion is read into evidence but is not
received as an exhibit (i.e., the jury never sees it).
D. CROSS-EXAMINATION
2. Scope of Cross-Examination
Although a party is entitled as of right to some cross-examination, the extent or scope of
cross-examination, like the order of calling witnesses, is frequently a matter of judicial
discretion. Cross-examination is hedged about by far fewer rules than is direct examina-
tion. On cross-examination, leading questions are permissible, as are, obviously, efforts at
impeachment. The most significant restriction is that the scope of cross-examination cannot
range beyond the subject matter of the direct examination. This restriction does not apply to
inquiries directed toward impeachment of the witness.
a. Restrictions on Scope
Under Federal Rule 611(b) and in the majority of American jurisdictions, cross-exami-
nation is limited to: (i) matters brought out on direct examination and the inferences
naturally drawn from those matters, and (ii) matters affecting the credibility of the
witness.
b. Significance of Restrictions
The question of the proper scope of cross-examination is important since it affects
the right to use leading questions. And, in jurisdictions that do not allow a party to
impeach his own witness, going beyond the scope of direct examination means that you
have made the witness “your own witness” and therefore cannot impeach his testimony.
Further, if the party placing a witness on the stand is the holder of a privilege, the court
may hold it waived to the extent that the other party may engage in cross-examination;
44. EVIDENCE
therefore, the scope of cross-examination permitted may determine to what extent the
cross-examining party may inquire into privileged material.
c. Collateral Matters
The general rule is that the cross-examiner is bound by the answers of the witness to
questions concerning collateral matters. Thus, the cross-examiner cannot refute the
response of the witness by producing extrinsic evidence. Indeed, some federal courts
resolve the matter under Rule 403 by treating the evidence on the collateral matter as
being substantially outweighed by time considerations and the danger of confusion of
the issues. Certain matters of impeachment, however, are recognized as sufficiently
important to merit development by extrinsic evidence (e.g., bias or interest of the
witness may be shown by other evidence, even if denied by the witness on cross-exami-
nation). Other matters of impeachment are limited solely to inquiry on cross-examina-
tion (e.g., prior misconduct of the witness not resulting in conviction but affecting the
witness’s credibility). Once beyond recognized impeachment techniques, the test as to
what is “collateral” is sufficiently vague to permit a wide exercise of discretion by the
trial judge.
E. CREDIBILITY—IMPEACHMENT
Impeachment means the casting of an adverse reflection on the veracity of the witness. The
primary method of impeachment is by cross-examination of the witness under attack, although
witnesses are often impeached by extrinsic proof that casts doubt on credibility. In terms of
relevance, any matter that tends to prove or disprove the credibility of the witness should be
admitted here.
1. Accrediting or Bolstering
a. General Rule—No Bolstering Until Witness Impeached
A party may not bolster or accredit the testimony of his witness until the witness has
been impeached.
Example: A prior statement made by W at the time of the event that is consistent
with W’s in-court testimony would not be admissible to show that W’s
memory of the event is excellent or that he told the same story twice and
therefore is especially worthy of belief.
b. Exceptions
The rule against accrediting is subject to exception where timeliness may raise an infer-
ence on the substantive issues of the case.
1) Timely Complaint
In certain cases a party may prove that the witness made a timely complaint, in
order to bolster the party’s credibility.
Examples: 1) Evidence of a prompt complaint of a rape victim is admissible
to bolster the complainant’s credibility in a subsequent criminal
prosecution.
2) Prior Identification
Evidence of any prior statement of identification made by a witness is admissible
not only to bolster the witness’s testimony, but also as substantive evidence that the
identification was correct. [Fed. R. Evid. 801(d)(1)(C); and see VII.B.1., infra]
2. Any Party May Impeach
Contrary to the traditional rule, under which a party could not impeach his own witness,
the Federal Rules provide that the credibility of a witness may be attacked by any party,
including the party calling him. [Fed. R. Evid. 607] Even under the traditional rule, however,
a party could impeach his own witness if the witness: (i) was an adverse party, (ii) was hostile,
(iii) was one required to be called by law, or (iv) gave damaging surprise testimony.
3. Impeachment Methods—Cross-Examination and Extrinsic Evidence
A witness may be impeached either by cross-examination (by eliciting facts from the witness
that discredit his own testimony) or by extrinsic evidence (by putting other witnesses on the
stand who will introduce facts discrediting his testimony).
Note: In the discussion that follows, “cross-examination” also includes a party impeaching
his own witness on direct examination.
There are certain well-recognized, often-used impeachment methods. These traditional
impeachment devices include: the use of prior inconsistent statements; a showing of bias or
interest in the litigation; an attack on the character of the witness by showing convictions
of crime, prior acts of misconduct, or poor reputation for veracity; a showing of sensory
deficiencies; and proof of contradictory facts. The basic questions for each of these methods
are: Is the examiner limited to impeachment by cross-examination alone, or may he produce
extrinsic evidence? If extrinsic evidence is permissible, must a foundation first be laid by
inquiry on cross-examination?
a. Prior Inconsistent Statements
For the purpose of impeaching the credibility of a witness, a party may show that the
witness has, on another occasion, made statements that are inconsistent with some
material part of his present testimony. Under the Federal Rules, an inconsistent state-
ment may be proved by either cross-examination or extrinsic evidence. To prove the
statement by extrinsic evidence, certain requirements must first be met: (i) a proper
foundation must be laid; and (ii) the statement must be relevant to some issue in the
case, i.e., it cannot be a “collateral matter.”
1) Laying a Foundation
Generally, extrinsic evidence of the witness’s prior inconsistent statement is admis-
sible only if: (i) the witness is, at some point, given an opportunity to explain or
deny the allegedly inconsistent statement; and (ii) the adverse party is, at some
point, given an opportunity to examine the witness about the statement. (These
opportunities need not come before introduction of the statement under the Federal
Rules.) This foundation requirement may be dispensed with, however, where
“justice so requires” (as where the witness has left the stand and is not available
when his prior inconsistent statement is discovered). Furthermore, this founda-
tion is not required when the prior inconsistent statement qualifies as an opposing
party’s statement (see VII.B.2., infra). [Fed. R. Evid. 613(b)] Also, the courts
generally agree that inconsistent statements by a hearsay declarant may be used
46. EVIDENCE
to impeach the declarant despite the lack of a foundation (obviously, where the
declarant is not a witness no foundation could be laid anyway). [Fed. R. Evid. 806]
b. Bias or Interest
Evidence that a witness is biased or has an interest in the outcome of a suit tends to
show that the witness has a motive to lie. A witness may always be impeached by
extrinsic evidence of bias or interest, provided a proper foundation is laid. Note that
evidence that is substantively inadmissible may be admitted for impeachment purposes
if relevant to show bias or interest.
Examples: 1) It may be shown that a witness is being paid to testify, that a witness
is financing the case, or that he otherwise has a financial interest in the
outcome of the litigation.
1) Foundation
Most courts require that before a witness can be impeached by extrinsic evidence
of bias or interest, he must first be asked about the facts that show bias or interest
on cross-examination. If the witness on cross-examination admits the facts claimed
to show bias or interest, it is within the trial judge’s discretion to decide whether
extrinsic evidence may be introduced as further proof of bias or interest.
c. Conviction of Crime
Under certain circumstances, a witness may be impeached by proof of conviction of a
crime. [Fed. R. Evid. 609] The fact that the witness (including a defendant who testifies
EVIDENCE 47.
in a criminal case) has been convicted of a crime may usually be proved by either
eliciting an admission on direct or cross-examination or by the record of conviction.
2) Type of Crime
a) Crime Involving Dishonesty or False Statement
Under the Federal Rules, a witness’s character for truthfulness may be
attacked (or impeached) by any crime (felony or misdemeanor) if it can be
readily determined that conviction of the crime required proof or admis-
sion of an act of dishonesty or false statement. Federal courts interpret this
category narrowly to encompass only offenses in the nature of crimen falsi
(e.g., perjury, false statement, criminal fraud, embezzlement, false pretense).
In most cases, the statutory elements will indicate whether an act of dishon-
esty or false statement was required. An indictment, statement of admitted
facts, or jury instructions may also be used to show that the crime required
proof of such an act. The trial court has no discretion—not even under
Federal Rule 403—to disallow impeachment by such crimes.
b) Felony Not Involving Dishonesty or False Statement
A witness’s character for truthfulness may also be attacked, under the Federal
Rules, by any felony whether or not it involves dishonesty or a false state-
ment. However, if the felony is one that does not involve dishonesty or false
statement, the trial court may exercise discretion to exclude it under one of the
following standards.
though the witness was never convicted. Federal Rule 608 permits such inquiry, in
the discretion of the court, only if the act of misconduct is probative of truthful-
ness (i.e., is an act of deceit or lying).
1) By Proof of Reputation
A witness may be impeached by showing that she has a poor reputation for truth-
fulness. The usual method of impeachment is to ask other witnesses about her
general reputation for truth and veracity in the community in which she lives. The
modern view is to allow evidence of reputation in business circles as well as in the
community in which the witness resides.
2) By Opinion Evidence
Most states do not allow the impeaching witness to state her opinion as to the
character of a witness for truth and veracity. However, the Federal Rules allow an
impeaching witness to state her personal opinions, based upon acquaintance, as to
the truthfulness of the witness sought to be impeached. [Fed. R. Evid. 608(a)]
f. Sensory Deficiencies
A witness may be impeached by showing that he had no knowledge of the facts to
which he testified, or that his faculties of perception and recollection were so impaired
50. EVIDENCE
as to make it doubtful that he could have perceived those facts. Such a showing can be
made either on cross-examination or by the use of extrinsic evidence.
1) Defects of Capacity
a) Perceptive Disabilities
It is, of course, proper to show deficiencies of the senses, such as deafness or
color blindness, that would have substantially impaired the witness’s ability to
perceive the facts to which he testifies. It may also be shown that at the time
the witness observed the events his perception was temporarily diminished
(e.g., that he was sleepy or under the influence of alcohol or drugs).
b) Lack of Memory
A witness can be impeached by showing that he has a poor memory of the
events about which he testifies. This is usually done on cross-examination by
asking the witness about other related matters to suggest the inference that if
his memory of related matters is poor, his recollection of the events to which
he is testifying is doubtful.
c) Mental Disorders
Psychiatric evidence of a mental disorder that would affect a witness’s credi-
bility has been admitted by some courts (particularly in sex offense cases).
2) Lack of Knowledge
a) Expert Witnesses
The credibility of an expert witness may be attacked by cross-examining him
as to (i) his general knowledge of the field in which he is claiming to be an
expert, and (ii) his particular knowledge of the facts upon which his opinion
is based.
b) Opinion Witnesses
The credibility of an opinion witness may be attacked by showing lack of
knowledge. For example, a witness who gives opinion evidence on the value
of land may be cross-examined regarding her knowledge of land values and
may be asked about sales of other land.
c) Character Witnesses
As discussed earlier, when a character witness testifies to the good character
of another (e.g., a defendant), the witness may be cross-examined regarding
the basis of his statement that the defendant’s character is good. In other
words, the testimony of the character witness may be discredited by asking
him about specific criminal or immoral acts committed by the defendant,
on the theory that if the witness has no knowledge of these acts, he does not
really know the defendant’s character. [Fed. R. Evid. 405(a)]
“Have you heard that the defendant . . . ?” However, under the Federal Rules
and in modern jurisdictions that permit character witnesses to testify as to
their opinions of character, questions in the form, “Do you know . . . ?”
would be proper.
g. Contradictory Facts
Extrinsic evidence of facts that contradict a witness’s testimony may sometimes be
admitted to suggest that a witness’s mistake or lie on one point indicates erroneous or
false testimony as to the whole. Extrinsic evidence of contradictory facts to impeach
is permitted: (i) where the witness’s testimony on a particular fact is a material issue
in the case, (ii) where the testimony on a particular fact is significant on the issue of
credibility, or (iii) where the witness volunteers testimony about a subject as to which
the opposing party would otherwise be precluded from offering evidence. However,
as discussed in 4., below, extrinsic evidence is not permitted to prove contradictory facts
that are collateral, i.e., not relevant to any material issue in the case or insignificant on
the issue of credibility.
Example: A defendant charged with cocaine possession testifies that he has never
in his life handled or come into contact with illegal narcotics of any
kind. The defendant could be impeached with extrinsic evidence that he
was seen purchasing marijuana on two occasions prior to the cocaine
incident.
against whom the out-of-court statement was offered may call the declarant as a witness and
cross-examine him about the statement.
6. Rehabilitation
A witness who has been impeached may be rehabilitated on redirect examination or by
extrinsic evidence.
a. Explanation on Redirect
For purposes of rehabilitation, the witness on redirect examination may explain or
clarify facts brought out on cross-examination.
Example: A witness testifying for the prosecution in an organized crime murder
trial admitted to making a prior inconsistent statement. On redirect, the
witness is permitted to explain that he gave a prior untruthful statement
favoring the defendant out of fear of being killed by the defendant’s
gang.
2) Exceptions
1. Objections
Unless an objection is made by opposing counsel, almost any kind of evidence will be
admitted. Failure to object is deemed a waiver of any ground for objection. The trial judge
need not raise grounds for objection on her own, but may take notice of plain errors affecting
substantial rights (e.g., admission of coerced confession not objected to by defense).
deposition, thereby affording counsel an opportunity to correct the form of his question.
An objection based on a testimonial privilege should also be made then, lest it be
deemed waived. However, objections going to the substance of a question or answer
(e.g., relevance, hearsay) can be postponed until the deposition is offered in evidence.
c. Specificity of Objections
An objection may be either general (“I object”) or specific (“Object, hearsay”). The
importance of whether an objection is general or specific lies in the extent to which
each type preserves the evidentiary issue on appeal. The following rules apply:
3. Offers of Proof
On some occasions, error cannot be based on exclusion of evidence unless there has been an
“offer of proof” that discloses the nature, purpose, and admissibility of the rejected evidence.
There are three types of “offers of proof.”
a. Witness Offer
Subsequent to a sustained objection by opposing counsel, the examining counsel
proceeds with his examination of a witness on the stand, out of the jury’s hearing, thus
making his record by the question-and-answer method.
b. Lawyer Offer
Counsel himself states, in narrative form, what the witness would have testified had he
been permitted to do so. The “witness offer” is generally preferred to the “lawyer offer”
and can be required by the trial court, especially if opposing counsel denies that the
witness would testify as narrated.
c. Tangible Offer
A marked, authenticated, and offered item of tangible evidence is its own offer of proof.
G. TESTIMONIAL PRIVILEGES
Testimonial privileges, which permit one to refuse to disclose and prohibit others from disclosing
certain sorts of confidential information in judicial proceedings, have two basic reasons for their
existence: (i) practicality, and (ii) society’s desire to encourage certain relationships by ensuring
their confidentiality, even at the high price of losing valuable information.
Some of the testimonial privileges are frankly grounded on hardheaded practicality. The
particular kind of disclosure could not be obtained, as a practical matter, even if there were no
privilege. Many priests, even when confronted by a contempt of court citation, would refuse to
breach the priest-penitent relationship. Society values some relationships sufficiently that it is
willing to protect their confidential nature even at the expense of the loss of information relevant
to the issues of a lawsuit. These relationships will be encouraged if confidentiality, when desired,
is assured. To put it more concretely, persons might forgo needed medical attention or be less
than candid with legal counsel were there no guarantee that communications made during the
physician-patient and attorney-client relationships would be accorded confidential status in legal
proceedings.
2. General Considerations
56. EVIDENCE
b. Confidentiality
To be privileged, a communication must be shown to have been made in confidence.
Many states, however, recognize a presumption that any disclosures made in the course
of a relationship for which a privilege exists were made in confidence.
d. Waiver
All types of privileges are waived by the following:
(i) Failure to claim the privilege by the holder herself or failure to object when privi-
leged testimony is offered;
(ii) Voluntary disclosure of the privileged matter by the holder (or someone else with
the holder’s consent) unless the disclosure is also privileged; or
A privilege is not waived where someone wrongfully disclosed information without the
holder’s consent. Similarly, a waiver of the privilege by one joint holder does not affect
the right of another joint holder to claim the privilege.
e. Eavesdroppers
A privilege based on confidential communications is not abrogated because the commu-
nication is overheard by someone whose presence is unknown to the parties; i.e., the
privilege still applies to the parties to the confidential communication. There is some
question, however, as to whether the eavesdropper may testify. The traditional view is
that the eavesdropper may testify to what he has overheard. But a significant number
of modern cases and statutes assert that as long as the holder of the privilege was not
negligent, there is no waiver of the privilege, and the eavesdropper is also prohibited
from testifying.
3. Attorney-Client Privilege
The first testimonial privilege ever established was the attorney-client privilege. It is a
common law privilege, although in some jurisdictions it has now been codified by statute. It
carries with it fewer exceptions than any other privilege.
a. Attorney-Client Relationship
The attorney-client privilege requires that the attorney-client relationship exist at the
time of the communications. The client, or her representative, must be seeking the
professional services of the attorney at the crucial time. But note that retainer negotia-
tions, involving disclosures made before the attorney has decided to accept or decline
the case, are covered if the other requirements of the privilege are present.
1) Client
A “client,” in the context of the typical formulation of the attorney-client privilege,
can be an individual private citizen, a public officer, a corporation, or any other
organization or entity, public or private, seeking professional legal services.
2) Representative of Client
A “representative of a client” is one having the authority to obtain legal services
or to act on advice rendered by an attorney, on behalf of the client.
3) Attorney
An “attorney” is any person who is authorized or, in many jurisdictions, who is
reasonably believed by the client to be authorized, to practice law in any state or
nation.
4) Representative of Attorney
A “representative of an attorney” is one employed by the attorney to assist in the
rendition of professional services, e.g., a clerk or secretary.
5) Corporation as Client
A corporation, as indicated above, can be a “client” within the meaning of the
attorney-client privilege. The statements of any corporate officials or employees
made to the attorney are protected if they were authorized or directed by the
corporation to make such statements.
b. Confidential Communication
A communication is “confidential” if it was not intended to be disclosed to third
persons, other than those to whom disclosure would be in furtherance of the rendi-
tion of legal services to the client or those who are necessary for the transmission of
the communication. Communications made in the known presence and hearing of a
stranger are not privileged.
1) Communications Through Agents
Communications made to third persons are confidential, and thus covered by the
attorney-client privilege, if necessary to transmit information between the attorney
and client. Examples include: communications by the client to the attorney’s
secretary or messenger; information (not documents) given to the attorney by the
client’s accountant; communications between the client’s liability insurer and the
attorney; and communications through an interpreter.
58. EVIDENCE
a) Examination by Doctor
When a client is examined by a doctor at the attorney’s request, the commu-
nications involved between the client and doctor (and the doctor and attorney)
are not covered by the physician-patient privilege because no treatment is
contemplated. These communications are, however, covered by the attorney-
client privilege because the examination is necessary to help the client
communicate her condition to the attorney. Note that this privilege would be
waived if the doctor were later called as an expert witness by the same client.
Example: P, a pedestrian, was struck by a car driven by D. P employs
Attorney to bring a negligence suit against D to recover for the
physical injuries P suffered in the accident. Attorney sends P
to Dr. Z for an evaluation of the extent and permanence of P’s
injuries. Attorney does not intend to call Dr. Z as an expert
witness at trial. At trial, D’s attorney, believing that P may
have admitted to some negligence of his own when describing
his injury, calls Dr. Z to testify to his examination of P. If P
objects, claiming attorney-client privilege, he may prevent Dr.
Z from testifying.
2) No Privilege Where Attorney Acts for Both Parties
Where an attorney acts for both parties to a transaction, no privilege can be
invoked in a lawsuit between the two parties (they obviously did not desire and
could not expect confidentiality as between themselves in a joint consultation), but
the privilege can be claimed in a suit between either or both of the two parties and
third persons (multiple parties can desire and expect confidentiality as against the
outside world).
c. Client as Holder of Privilege
The privilege, if it exists, can be claimed by the client, her guardian or conservator,
the personal representative of a deceased client, or the successor, trustee, or similar
representative of a corporation, association, etc. The person who was the attorney at
the time of the communication can claim the privilege, but only on behalf of the client.
The attorney’s authority to do this is presumed in the absence of any evidence to the
contrary.
d. Duration of Privilege
The attorney-client privilege applies indefinitely. Termination of the attorney-client
relationship does not terminate the privilege. The privilege even continues to apply
after the client’s death. Rationale: Knowing that communications will remain confiden-
tial even after death encourages the client to communicate fully and frankly with her
attorney. [Swidler & Berlin v. United States, 524 U.S. 399 (1998)]
4. Physician-Patient Privilege
The physician-patient privilege is a statutory privilege, which has not been adopted in all
60. EVIDENCE
2) In Aid of Wrongdoing
Like the attorney-client privilege, there is no privilege if the physician’s services
were sought or obtained in aid of the planning or commission of a crime or tort, or
to escape detection or apprehension after the commission of a crime or tort.
c. Criminal Proceedings
There is a split of authority as to the applicability of the physician-patient privilege in
criminal proceedings. In some states, the privilege applies in both civil and criminal
cases. In a number of others, it cannot be invoked in criminal cases generally. In still
other states, the privilege is denied in felony cases, and in a few states, it is denied only
in homicide cases. Note that where a psychiatrist is involved, however, the applicable
privilege is the psychotherapist-client privilege (below), which is more widely accepted
in all proceedings than the physician-patient privilege.
a) Marital Relationship
The communication must be made during a valid marriage. Divorce will not
terminate the privilege retroactively, but communications after divorce are not
privileged.
c. Nonapplicability of Privileges
Neither the spousal immunity nor the confidential marital communications privilege
applies in actions between the spouses or in cases involving crimes against the testi-
fying spouse or either spouse’s children (e.g., assault and battery, incest, bigamy, child
abuse, etc.).
a. “Incriminating” Defined
Testimony is incriminating if it ties the witness to the commission of a crime or would
furnish a lead to evidence tying the witness to a crime; the testimony need not prove
guilt. [Hoffman v. United States, 341 U.S. 479 (1951)] A witness cannot refuse to
answer because of exposure to civil liability; it must be to avoid criminal liability.
8. Clergy-Penitent Privilege
A person has a privilege to refuse to disclose, and to prevent others from disclosing, a confi-
dential communication by the person to a member of the clergy in the clergy member’s
capacity as a spiritual adviser. A member of the clergy can be a minister, priest, rabbi, or
other similar functionary of a religious organization, or reasonably believed to be so by the
person consulting him. This common law privilege is very similar in its operation to the
attorney-client privilege, supra.
9. Accountant-Client Privilege
This is a statutory privilege, found in a number of jurisdictions, which is similar to the
attorney-client privilege, supra.
Less than half of the states have enacted statutes protecting the journalist’s source of infor-
mation, and the protection ranges from an absolute privilege to one qualified by the need for
disclosure in the public interest.
64. EVIDENCE
a. Identity of Informer
The federal government, or a state or subdivision of a state, generally has a privilege
to refuse to disclose the identity of a person who has furnished to a law enforcement
officer information purporting to reveal the commission of a crime.
b. Official Information
This is a general catch-all privilege that attaches to certain communications made by or
to public officials. Official information has been defined as information not open to the
public, relating to the internal affairs of the government or its subdivisions. It applies to
some fairly low-level communications made by or to officials (e.g., a judge’s communi-
cations to his law clerk).
testifying at the current trial or hearing, offered in evidence to prove the truth of the matter
asserted. [Fed. R. Evid. 801(c)] The rule against hearsay is probably the most important exclu-
sionary rule of evidence. If a statement is hearsay, and no exception to the rule is applicable, the
evidence must be excluded upon appropriate objection to its admission. [Fed. R. Evid. 802]
a. Cross-Examination of Declarant
Note that it is the declarant who made the statement that the adverse party was not able
to cross-examine. Of course, the adverse party can cross-examine the witness who
repeats the statement, but this does not help much where all the witness does is repeat a
statement as to which the party needs to cross-examine the original declarant.
2. “Statement”
For purposes of the hearsay rule, a “statement” is a person’s (i) oral or written assertion, or
(ii) nonverbal conduct intended as an assertion. [Fed. R. Evid. 801(a)]
a. Oral Statements
“Statement” includes oral statements (i.e., where the witness testifies that somebody
said “ . . . ”).
b. Writings
Any written document that is offered in evidence constitutes a “statement” for hearsay
purposes.
c. Assertive Conduct
Conduct intended by the actor to be a substitute for words (e.g., the nod of the declar-
ant’s head indicating yes) is a “statement” within the meaning of the hearsay rule.
66. EVIDENCE
d. Nonassertive Conduct
Under the traditional common law definition of hearsay, “statement” included nonas-
sertive conduct—sometimes called “Morgan hearsay.” Nonassertive conduct is conduct
the declarant did not intend as an assertion but which is being offered as an assertion.
However, under modern codes and the Federal Rules, evidence of nonassertive conduct
is not hearsay. The rationale is that the likelihood of fabrication is less with nonassertive
conduct than with assertive or verbal conduct.
Examples: 1) Consider the conduct of a deceased sea captain who, after examining
every part of a ship, embarked on it with his family, when his conduct
is being introduced on the question of the seaworthiness of the vessel.
Although the sea captain did not intend his embarking on the vessel to
serve as an assertion of anything, his conduct can be used to imply that
he thought the ship was seaworthy and, since he knew his business, that
the ship was in fact seaworthy.
2) Similarly, the fact that a doctor treated a person for plague is nonas-
sertive conduct by the doctor that could be used to show that the person
had plague.
The following are other examples of out-of-court statements that are not hearsay.
distinction makes little difference because the result (admissibility of the state-
ments) is the same.
4. Nonhuman Declarations
There is no such thing as animal or machine hearsay. Hearsay involves an out-of-court state-
ment by a person. Therefore, a witness who testifies to the time of day (what the clock says)
or to radar readings (what the machine says) is not testifying to hearsay. Data that is gener-
ated completely electronically is not hearsay. Similarly, the behavior of a drug-sniffing dog
in identifying a suspect is not hearsay. The issues presented by these examples are ones of
relevance or reliability of the mechanism or animal. For example, a witness may testify to
the actions of a drug-sniffing dog in identifying a suspect if there is a foundation showing
that the dog was properly trained and is reliable in identifying drug carriers.
a. Hearsay
1) On the issue of whether the traffic light was red or green, the witness testifies that
he was told by Decla that the light was green. (Oral hearsay.)
3) On the issue of whether Spano had been a resident of New York for one year prior
to commencing his lawsuit, Spano offers the affidavit of Decla that Spano had
lived in Buffalo for 10 years. (Written hearsay; under oath, but hearsay nonethe-
less.)
4) On the issue of whether Yuckl was the child molester, a police officer testifies
that when he asked the child-victim whether the perpetrator had a beard, the child
nodded his head. (Hearsay by assertive conduct; nodding, which translates, “Yes,
the man had a beard.”)
5) On the issue of whether the painting sold to Harvey was actually a genuine
Picasso, there is offered a dealer’s bill of sale describing the painting as a Picasso.
(Written hearsay.)
b. Nonhearsay
in fact a Picasso. The evidence, in other words, was offered to show the impact of
the dealer’s representation on the defendant’s state of mind, i.e., his belief.)
3) On the issue of whether landlord knew about a defective stair, a witness testifies
that he heard Decla say to the landlord, “The stair is broken.” (Offered to prove
notice, not that the stair was in fact broken.)
4) On the issue of whether the complaining witness had a venereal disease, Grutz
testifies for the prosecution that the complaining witness had not been placed in the
venereal disease ward upon her admission to the girls’ reformatory. (Nonhearsay
under the Federal Rules, since it is nonassertive conduct.)
5) On the issue of whether a transfer of a share of stock from Decla to Bushmat was
a sale or a gift, Bushmat testifies that Decla made a statement at the time of the
transfer: “I’m giving you this share of stock as a birthday present.” (Legally opera-
tive words of gift.)
6) Action for personal injuries by a guest in an automobile against its owner. On the
issues of contributory negligence and assumption of risk, a witness testifies that an
hour before the accident a mechanic said to the owner in the presence of the guest,
“The tread on that left front tire is paper-thin. You’re likely to have a blowout.”
(Notice, knowledge; not offered to establish that in truth the tread was thin.)
7) Action of P against D. Witness No. 1 testifies for P that D’s car was going “over 70
miles an hour.” To impeach Witness No. 1, D offers the testimony of Witness No.
2 that Witness No. 1 said a day after the accident that D was going “slowly.” (Used
solely to cast doubt on credibility; not offered to establish the truth of the asser-
tion.)
(see D.5., infra), but Anne’s incorporated statement does not fall within any
hearsay exception. Thus, the report is inadmissible.
Note: Although an opposing party’s statement will fall within this hearsay exclusion even if
it does not “admit” anything, these statements are still commonly referred to as “admissions”
by courts and commentators. Because the term “admissions” better distinguishes these state-
ments from statements against interest (an exception to the hearsay rule, see C.3., infra), this
outline will generally refer to these statements as admissions.
EVIDENCE 71.
a. In General
A statement need not have been against interest at the time it was made to qualify as an
admission (compare the statement against interest exception, C.3., infra). The statement
may even be in the form of an opinion. Note that some states admit these statements
under an exception to the hearsay rule.
3) Adoptive Admissions
A party may expressly or impliedly adopt someone else’s statement as his own,
thus giving rise to an adoptive admission. [Fed. R. Evid. 801(d)(2)(B)]
Example: Where Plaintiff claims an orthopedic abnormality in a suit
against Defendant, Defendant may properly offer against Plaintiff
Plaintiff’s prior application for a chauffeur’s license, which included
a doctor’s certificate stating that Plaintiff had “no orthopedic abnor-
mality.”
a) Silence
If a party fails to respond to accusatory statements where a reasonable person
would have spoken up, his silence may be considered an implied admission.
For silence to be an admission the following requirements must be met:
(i) The party must have heard and understood the statement;
(ii) The party must have been physically and mentally capable of denying
the statement; and
(iii) A reasonable person would have denied the accusation under the same
circumstances.
b. Vicarious Admissions
An admission is frequently not the statement or act of the party against whom the
admission is offered at trial. The question that remains is—what relationship must exist
between the declarant and the party to make the former’s statement admissible against
the latter?
1) Co-Parties
Statements of a party are not receivable against her co-plaintiffs or co-defen-
dants merely because they happen to be joined as parties to the action. If there
are two or more parties, the admission of one is receivable against her but, in the
absence of authority, not against her co-party.
2) Authorized Spokesperson
The statement of a person authorized by a party to speak on its behalf (e.g., state-
ment by company’s press agent) can be admitted against the party. [Fed. R. Evid.
801(d)(2)(C)]
3) Principal-Agent
Statements by an agent or employee concerning any matter within the scope of her
agency or employment, made during the existence of the agency or employment
relationship, are admissible against the principal. [Fed. R. Evid. 801(d)(2)(D)]
Therefore, if a truck driver-employee has an accident while on the job and admits
that she was negligent, this admission may be introduced against her employer
even if she was not authorized to speak for the employer.
4) Partners
After a partnership is shown to exist, an admission of one partner, relating
to matters within the scope of the partnership business, is binding upon her
co-partners since, as to such matters, each partner is deemed the agent of the
others.
5) Co-Conspirators
The Supreme Court has held that admissions of one conspirator, made to a third
party in furtherance of a conspiracy to commit a crime or a civil wrong, at a
time when the declarant was participating in the conspiracy, are admissible against
co-conspirators. [United States v. Inadi, 475 U.S. 387 (1986)] The thought is that
a conspiracy is analogous to a partnership—“partners in crime.” The govern-
ment need not demonstrate the unavailability of a nontestifying co-conspirator as
a prerequisite to admission of the co-conspirator’s out-of-court statements under
Rule 801(d)(2)(E). Also, the court must use the co-conspirator’s statement itself,
together with other evidence, to determine whether the statement is admissible.
[Fed. R. Evid. 801(d)(2)] In other words, the proponent is not required to establish
the existence of the conspiracy, and the participation of the declarant and party,
with evidence that is entirely independent of the statement itself.
affected by admissions of the former owner made before the owner parted with her
interest. In most state courts, admissions of each joint owner are admissible against
the other, and admissions of a former owner of real property made at the time
she held title are admissible against those claiming under her (grantees, heirs,
devisees, or otherwise). These statements are not considered admissions under the
Federal Rules, although they may be admissible under one of the hearsay excep-
tions (e.g., as a statement against interest).
1. “Unavailability” Defined
A declarant is unavailable if:
(i) He is exempted from testifying because the court rules that a privilege applies;
(ii) He refuses to testify concerning the statement despite a court order to do so;
(v) He is absent (e.g., beyond the reach of the trial court’s subpoena) and the statement’s
proponent has been unable to procure his attendance or testimony by process or other
reasonable means.
[Fed. R. Evid. 804(a)(1) - (5)] Note that a declarant is not unavailable if his “unavailability”
was procured by the proponent of the statement or if the statement’s proponent did not
attempt to procure the declarant’s attendance.
2. Former Testimony
The testimony of a now unavailable witness given at a trial, hearing, or in a deposition taken
74. EVIDENCE
a. Identity of Parties
The requirement of identity of parties does not mean that parties on both sides of the
controversies must be identical. It requires only that the party against whom the testi-
mony is offered or, in civil cases, the party’s predecessor in interest was a party in the
former action. “Predecessor in interest” includes one in a privity relationship with the
party, such as grantor-grantee, testator-executor, life tenant-remainderman, and joint
tenants. The requirement of identity of parties is intended merely to ensure that the
party against whom the testimony is offered (or a predecessor in interest in a civil case)
had an adequate opportunity and motive to cross-examine the witness.
d. Under Oath
The former testimony must have been given under oath or sworn affirmation.
1) The accused or his attorney was present and had the opportunity to cross-
examine at the time the testimony was given (e.g., at a preliminary examination or
a former trial for the same offense); and
EVIDENCE 75.
1) The statement must have been against pecuniary, proprietary, or penal interest
when made, such that a reasonable person in the declarant’s position would have
made it only if she believed it to be true.
3) Declarant must have been aware that the statement is against her interest and
she must have had no motive to misrepresent when she made the statement.
confession of a third party where to do so would deprive the accused of a fair trial.
[Chambers v. Mississippi, 410 U.S. 284 (1973)]
Note that under the traditional view, still followed by some states, the declaration was admis-
sible only in homicide prosecutions (not civil actions), and then only if the declarant actually
died.
a. Rationale
The rationale is that (i) insofar as the declarant knows her own state of mind, there is no
need to check her perception; (ii) since the statement is of present state of mind, there is
no need to check her memory; and (iii) since state of mind is in issue, it must be shown
some way—and very often, the declarant’s own statement is the only way.
78. EVIDENCE
b. When Admissible
2. Excited Utterances
A declaration made by a declarant during or soon after a startling event is admissible. The
declaration must be made under the stress of excitement produced by the startling event.
The declaration must concern the immediate facts of the startling occurrence. [Fed. R. Evid.
803(2)] The spontaneousness of such a declaration and the consequent lack of opportunity
for reflection and deliberate fabrication provide an adequate guarantee of its trustworthiness.
the most important factor in determining whether the declaration was made under the
stress of the excitement. If a declaration is made while the event is still in progress, it is
easy to find that the excitement prompted the utterance. Declarations made shortly after
the event have sometimes been excluded as mere narrative of past events. But when the
declaration is made so near to the time of the occurrence as to negate any probability of
fabrication, it is usually admissible.
b. Safeguards
Such a comment regarding a situation then before the declarant, i.e., the statement of a
present sense impression, does not have the supposed safeguards of impulse, emotion,
or excitement, but there are other safeguards of reliability. Statements of present
sense impression are safe from defects in memory. There is usually little or no time
for calculated misstatement. The statement will usually have been made to another
person—the very witness who reports it—who would have equal opportunity to
observe and to contradict or correct a misstatement.
Example: Decla said to N, “Look at that car go.” W may testify that Decla made
the statement in order to prove that the car was speeding.
also of the cause or source of the condition insofar as reasonably pertinent to diagnosis
or treatment. The statement need not be made to a physician. Statements to ambulance
drivers, hospital staff, or even the declarant’s family members might be included as
long as they are made for, and reasonably pertinent to, medical diagnosis or treatment.
Moreover, contrary to the majority state view, Rule 803(4) permits such declarations
even when made to a doctor employed to testify.
a. Rationale
The rationale for this exception lies in the belief that special reliability is provided by
the regularity with which business records are kept, their use and importance in the
business, and the incentive of employees to keep accurate records or risk employment
penalties. If a record qualifies as a business record, it may be admitted without calling
the author of the record or the employee with personal knowledge of the recorded event.
It makes no difference that the record is self-serving and offered in evidence by the
party whose business made the record.
1) “Business”
Although traditionally called a “business record,” the exception applies to records
kept by any “business, organization, occupation, or calling, whether or not for
profit.” Thus, the definition would include records made by churches, hospitals,
schools, etc. [Fed. R. Evid. 803(6)]
a) Business Activity
The record must have been maintained in conjunction with a business activity.
3) Personal Knowledge
The business record must consist of matters within the personal knowledge of
the entrant or within the personal knowledge of someone with a business duty to
transmit such matters to the entrant.
the informant or the recorder, even though the recorder lacked personal
knowledge. The integrity of the special reliability assumption is maintained
because the informant was under a business duty to report accurately and the
recorder was under a business duty to properly record the information.
Normally, the original or first permanent record of the transaction must be intro-
duced, but where the records to be introduced are voluminous, summaries or
compilations may be admitted.
(ii) Matters observed pursuant to a duty imposed by law (excluding police obser-
vations in criminal cases); or
(iii) In civil actions and proceedings and against the government in criminal
cases, factual findings (including opinions and conclusions) resulting from an
investigation made pursuant to authority granted by law.
[Fed. R. Evid. 803(8); Beech Aircraft Corp. v. Rainey, 488 U.S. 153 (1988)]
Examples: 1) A manual prepared by the office that processes Medicare claims,
explaining which claims are properly payable under Medicare, is
admissible against the defendant in a Medicare fraud case under
Federal Rule 803(8)(A).
a) Duty to Record
The writing must have been made by, and within the scope of duty of, the
public employee.
d. Judgments
A certified copy of a judgment is always admissible proof that such judgment has been
entered. The problem is to what extent the facts adjudicated in the former proceeding
can be introduced to prove facts in the present case.
of proving the facts upon which a conviction is based when such facts are
independently admissible either to prove specific acts of misconduct on the
issue of a person’s motive, intent, absence of mistake, etc., or as proof of prior
acts of sexual assault or child molestation in cases alleging sexual assault or
child molestation.
9. Learned Treatises
Many courts do not admit statements from standard scientific treatises or authoritative works
as substantive proof, limiting admissibility to use as impeachment of the qualifications of
the expert witness. The Federal Rules recognize an exception to the hearsay rule for learned
treatises. Federal Rule 803(18) provides for the substantive admissibility of a learned treatise
if the treatise is:
(i) Called to the attention of the expert witness upon cross-examination or relied upon by
her during direct examination; and
(ii) Established as reliable authority by the testimony or admission of the witness, by other
expert testimony, or by judicial notice.
Even under the Federal Rules, however, the relevant portion of the treatise is not actually
shown to the jury; it is admissible by being read into the record.
EVIDENCE 87.
10. Reputation
In addition to reputation testimony concerning a person’s character [Fed. R. Evid. 803(21)],
reputation evidence concerning someone’s personal or family history [Fed. R. Evid. 803(19)]
or concerning land boundaries or the community’s general history [Fed. R. Evid. 803(20)] is
admissible hearsay.
11. Family Records
Statements of fact concerning personal or family history contained in family Bibles, genealo-
gies, jewelry engravings, engravings on urns or burial markers, or the like are admissible
hearsay. [Fed. R. Evid. 803(13)]
12. Market Reports
Market reports and other published compilations (lists, directories, etc.) are admissible if
generally used and relied upon by the public or by persons in a particular occupation. [Fed.
R. Evid. 803(17)]
E. RESIDUAL “CATCH-ALL” EXCEPTION OF FEDERAL RULES
The Federal Rules provide a general catch-all exception for hearsay statements not covered
by specific exceptions. [Fed. R. Evid. 807] There are three requirements for a statement to be
admitted under the catch-all exception:
1. “Trustworthiness” Factor
First of all, the statement must have “circumstantial guarantees of trustworthiness” that are
equivalent to those of statements admitted under other hearsay exceptions.
2. “Necessity” Factor
The statement must be offered on a material fact, and must be more probative as to that fact
than any other evidence which the proponent can reasonably produce so that the “interests
of justice” will be served by its admission.
3. Notice to Adversary
Finally, the proponent must give notice in advance of trial to the adverse party as to the
nature of the statement (including the name and address of the declarant) so that the adver-
sary has an opportunity to prepare to meet it.
F. CONSTITUTIONAL ISSUES
1. The Confrontation Clause
In criminal cases, it may be argued that the use of hearsay evidence violates the accused’s
right to “confront” and cross-examine the witnesses against him. Note, however, that there
generally is no Confrontation Clause problem if the hearsay declarant is present at the trial
and is subject to cross-examination at that time.
a. Prior Testimonial Statement of Unavailable Witness
Under the Confrontation Clause, a hearsay statement will not be admitted (even if it
falls within a hearsay exception) when:
(i) The statement is offered against the accused in a criminal case;
(ii) The declarant is unavailable;
A. BURDENS OF PROOF
The term “burden of proof,” as used by judges and lawyers at trial, encompasses two separate
meanings or burdens. Thus, burden of proof can mean:
Plaintiff has made out a prima facie case of Defendant’s negligence. Put another way,
it means that Plaintiff has met her burden of going forward with evidence on the negli-
gence issue.
Now assume that Defendant rests immediately after Plaintiff’s case-in-chief without
producing any rebuttal evidence. Plaintiff then moves for a directed verdict in her favor,
claiming that Defendant was negligent as a matter of law. If this motion were granted,
it would mean that (i) Plaintiff’s evidence was sufficiently persuasive, (ii) the burden
shifted to Defendant, and (iii) Defendant failed to meet his newly imposed burden of
producing evidence of no negligence. If Plaintiff’s motion were denied, it would mean
only that Plaintiff met her initial burden but that it did not shift to Defendant; and,
therefore, the burden of production having dropped out of the case, the matter is now
for the jury. Once in the hands of the jury, the question is whether Plaintiff has met her
burden of persuasion.
B. PRESUMPTIONS
A presumption is a rule that requires that a particular inference be drawn from an ascertained set
of facts. It is a form of substitute proof or evidentiary shortcut, in that proof of the presumed fact
is rendered unnecessary once evidence has been introduced of the basic fact that gives rise to the
presumption. Presumptions are established for a wide variety of overlapping policy reasons. In
some cases, the presumption serves to correct an imbalance resulting from one party’s superior
access to the proof on a particular issue. In others, the presumption was created as a timesaver
to eliminate the need for proof of a fact that is highly probable in any event. In other words, the
inference from the basic fact to the presumed fact is so probable and logical that it is sensible to
assume the presumed fact upon proof of the basic fact. In still other situations, the presumption
serves as a social or economic policy device. It operates to favor one contention by giving it the
benefit of a presumption and to correspondingly handicap the disfavored adversary.
Example: A common presumption is that the driver of a vehicle is the owner’s agent. When
plaintiff has been injured by the negligent operation of a vehicle, the nondriving
owner-defendant will be liable for the negligence of the driver if the driver is the
owner’s agent. Plaintiff’s burden of proving that the driver was the agent of the
owner is aided by a presumption of agency that arises upon proof of ownership.
The justification of this presumption of agency from ownership may be explained
in terms of fairness in light of defendant’s superior access to the evidence; in
terms of probability, since it is unlikely that defendant’s car was stolen by the
driver; or in terms of a social policy of promoting safety or increasing available
compensation for traffic victims by widening the responsibility of owners.
2. Rebutting a Presumption
A presumption is overcome or destroyed when the adversary produces some evidence contra-
dicting the presumed fact. In other words, the presumption is of no force or effect when
sufficient contrary evidence is admitted. This is the federal view adopted by Federal Rule
301, except where state law provides the rule of decision (see 6., infra).
Example: Plaintiff-victim of automobile driver’s negligence sues owner. Plaintiff proves
92. EVIDENCE
ownership, thus giving rise to the presumption that the driver was the agent
of the owner. Defendant-owner testifies that the driver was not his agent, and
his evidence could justify a jury finding that the driver was without authority
from the owner. At this point, plaintiff’s presumption is gone. He will have
to sustain the burden of proving by a preponderance of the evidence that the
driver was the agent of the owner, or his case will fail.
a. Permissible Inferences
The permissible inference (prima facie case, or sometimes erroneously called
“presumption of fact”) will allow a party to meet the burden of production but will not
shift the burden to the adversary. Examples of situations giving rise to permissible
inferences are:
3) Undue Influence
Where the attorney who drafted a will is its principal beneficiary to the exclusion
of the natural objects of the testator’s bounty, an inference of undue influence may
be found.
a “presumed” fact against the accused. When the existence of a presumed fact is
submitted to the jury, the judge shall instruct the jury that it may regard the basic
facts as sufficient evidence of the presumed fact, but that it is not required by law
to do so. If the presumed fact establishes guilt, is an element of the offense, or
negates a defense, its existence must be found (proved) beyond a reasonable doubt.
c. Conclusive Presumptions
This form of inference goes beyond the true presumption since it cannot be rebutted by
contrary evidence. A “conclusive” presumption is really a rule of substantive law.
Example: In some states, it is conclusively presumed that a child under a certain
age (e.g., seven years old) cannot commit a crime. No evidence to the
contrary can rebut this presumption, and part of the proof of the case
requires a showing that the accused is over the minimum age.
4. Specific Presumptions
The following are common rebuttable presumptions:
a. Presumption of Legitimacy
The law presumes that every person is legitimate. The presumption applies to all cases
where legitimacy is in dispute. The mere fact of birth gives rise to the presumption. The
presumption is destroyed by evidence of illegitimacy that is “clear and convincing.” For
example, the presumption is overcome by proof of a husband’s impotency, proof of lack
of access, or the negative result of a properly conducted blood grouping test.
c. Presumption of Sanity
Every person is presumed sane until the contrary is shown. The presumption of sanity
applies in criminal as well as civil cases.
1) The person is inexplicably absent for a continuous period of seven years (death is
deemed to have occurred on the last day of the seven-year period); and
2) He has not been heard from, or of, by those with whom he would normally be
expected to communicate.
f. Presumption of Chastity
There is a presumption that every person is chaste and virtuous.
94. EVIDENCE
g. Presumption of Regularity
The general presumption is that no official or person acting under an oath of office will
do anything contrary to his official duty, or omit anything that his official duty requires
to be done.
h. Presumption of Continuance
Proof of the existence of a person, an object, a condition, or a tendency at a given time
raises a presumption that it continued for as long as is usual with things of that nature.
j. Presumption of Solvency
A person is presumed solvent, and every debt is presumed collectible.
l. Presumption of Marriage
Upon proof that a marriage ceremony was performed, it is presumed to have been
legally performed and that the marriage is valid. A presumption of marriage also arises
from cohabitation.
5. Conflicting Presumptions
If two or more conflicting presumptions arise, the judge shall apply the presumption that is
founded on the weightier considerations of policy and logic. For example, where the validity
of a later marriage is attacked by evidence of a prior valid marriage, the presumption of the
validity of the later marriage is deemed to prevail over the presumption of the continuance of
the first marriage.
1. Party Responsibility
Ours is an adversarial adjudicative process, and so the focus is on party responsibility or,
perhaps what is more to the point, on lawyer responsibility. Very little happens in the litiga-
tion process unless some lawyer makes it happen by filing pleadings and motions, by initi-
ating discovery, by entering into stipulations, by calling witnesses and offering exhibits at
trial, or by interposing objections to the admission of evidence. In other words, the parties,
through their lawyers, frame the issues in a litigation by making allegations, admissions, and
EVIDENCE 95.
denials in their pleadings, and by entering into binding stipulations. They assume the burden
of proving the issues they have raised. And then, by deciding which witnesses to call to the
stand and what tangible exhibits to introduce (and by deciding to what they will object), they
control the flow of evidence. But the parties and their lawyers are not the only ones to be
allocated important responsibilities in the adversary trial process.
2. Court-Jury Responsibility
Under our system, the trial court is more umpire than advocate. The trial judge’s primary
responsibility is to fairly superintend the trial; the judge is not permitted to become a
partisan in it. As a general rule, questions of law are for the trial court to deal with, and
questions of fact determination are for the jury, although trial judges frequently encounter the
necessity of making preliminary fact determinations in connection with such matters as the
admission or exclusion of evidence. (Of course, both types of questions—legal and factual—
are for the trial court in a nonjury case.)
introduction of evidence and then subsequently determines that a jury could not
reasonably find that the preliminary fact exists, she must instruct the jury to disre-
gard that evidence.
a) Agency
If plaintiff sues defendant upon an alleged contract, evidence of negotiations
with a third party is inadmissible because it is irrelevant unless the third party
is shown to be defendant’s agent. However, the evidence of the negotiations
with the third party is admissible if there is evidence sufficient to sustain a
finding of the agency.
b) Authenticity of a Document
If there is a dispute about whether a note was signed by the defendant (as
opposed to a forger), the authenticity of the document is to be left for the jury.
In a sense this is merely an issue of relevancy, since the note, if forged, is
irrelevant to the liability of the defendant.
c) Credibility
When a conviction of a crime is offered to attack the credibility of a witness,
the judge must admit the evidence and allow the jury to determine the
witness’s credibility if there is evidence sufficient to identify the witness as
the person convicted.
d) Personal Knowledge
The question of whether a witness had personal knowledge can go to the jury
if there is sufficient evidence to sustain a finding that the witness had personal
knowledge.
4) Others
The above two cases are the most important where the judge must first determine
the existence of a foundational fact. However, there are several other major catego-
ries.
b) Presence of Jury
Whether the jury should be excused during the preliminary fact determina-
tion is generally within the discretion of the trial judge. However, because of
the potential for prejudice to the accused in a criminal trial, the jury must be
excused during hearings on the “voluntariness” of the accused’s confession,
or whenever the accused testifies during the preliminary fact hearing and
requests that the jury be excused. [Fed. R. Evid. 104(c)]
f. Rulings
A trial judge has an obligation to rule promptly on counsel’s evidentiary objections and,
when requested to do so by counsel, to state the grounds for her rulings.
INTRODUCTORY NOTE
You can use the sample multiple choice questions below to review the law and practice your under-
standing of important concepts that you will likely see on your law school exam. To do more questions,
access StudySmart Law School software from the BARBRI website.
Question 1 Question 2
While driving home after an evening spent The defendant was on trial for the crime
drinking at a local bar, the plaintiff passed out of reckless homicide after allegedly running
at the wheel. His car went through a red light at over the victim with his car and killing him.
an intersection and was struck by a car driven The defendant testified that he was not driving
by the defendant. The plaintiff, under the influ- recklessly. Defense counsel wants to call to the
ence of alcohol, staggered from his car. The stand the defendant’s colleague, who regularly
defendant, believing that the plaintiff had been shares rides with the defendant to and from
injured in the accident, said “It’s my fault. I was work, to testify to his reputation for careful
not paying attention. I’ll take care of all your driving and for truthfulness.
medical bills.” Later that night, the plaintiff was
treated for minor injuries at a nearby hospital. Should the testimony be admitted?
The plaintiff sued the defendant for damages, (A) The testimony regarding the defendant’s
alleging that the defendant was driving negli- reputation as a careful driver should be
gently at the time of the accident. The plaintiff excluded, but the testimony regarding his
offered the testimony of a witness who was truthfulness should be admitted.
prepared to testify that, after the accident, the
defendant stated in a clear, calm voice, “I was (B) The testimony regarding the defendant’s
not paying attention. I’ll take care of all your reputation as a careful driver should be
medical bills.” admitted, but the testimony regarding his
truthfulness should be excluded.
Assuming the proper objection, should the
witness’s testimony concerning the defendant’s (C) The testimony should be excluded in its
statement be admitted? entirety.
(A) No, because the defendant’s statement is a (D) The testimony should be admitted in its
settlement offer. entirety.
Question 3 Question 4
At the defendant’s trial for armed robbery, the During the defendant’s prosecution for
prosecutor offers indisputable evidence tending robbery, the prosecutor asks the court to take
to show that the defendant committed two judicial notice of the fact that at that latitude, the
other armed robberies in the year preceding the sun is still up at 5:30 p.m. on June 21. The court
present offense, and that he committed all three so finds.
robberies to obtain money for his heroin habit.
The defendant has no prior convictions and has What is the effect of the court’s action?
chosen not to take the stand in his defense.
(A) The burden of persuasion is now on the
Should the court admit this evidence over the defendant to prove otherwise as to the fact
defendant’s objection? judicially noticed.
(A) No, because the defendant was not con- (B) The fact judicially noticed is established
victed of the other robberies. beyond a reasonable doubt.
(B) No, because the defendant has not testified (C) The prosecutor’s burden of producing
at his trial. evidence on the fact judicially noticed is
satisfied.
(C) Yes, unless the court determines that the
probative value of the evidence is substan- (D) The fact judicially noticed is conclusively
tially outweighed by its prejudicial effect. established.
Question 5 Question 6
The plaintiff brought a breach of contract suit The plaintiff sued the defendant for injuries
against the defendant, alleging that the defen- resulting from a car accident. At trial, the plain-
dant paid for only 25 cases of baseball cards tiff’s witness testified that the defendant ran a
even though the order was for 50. The defendant red light and then hit the plaintiff’s car.
claimed that only 25 cases were delivered to his
store. The plaintiff then introduced a shipping Which of the following is the court most
bill from a freight company showing that pickup likely to allow to impeach the credibility of the
had been made from the plaintiff on 50 cases. witness?
May the defendant now compel the plaintiff to (A) A record of a juvenile offense.
introduce the remainder of the record, showing
that the freight company had only 25 cases of (B) A question on cross-examination asking
baseball cards on its truck? whether the witness lied on a recent job
application.
(A) Yes, after a proper foundation is laid.
(C) A certified copy of a conviction for robbery
(B) Yes, because fairness dictates that it should five years ago for which the witness has
be considered contemporaneously with since been proved innocent and pardoned.
what the plaintiff is offering.
(D) A record of an arrest one week ago for
(C) No, because the information is irrelevant to criminal trespass.
the case.
Question 7 Question 8
The plaintiff is suing the defendant for injuries The plaintiff sued the defendant, the owner
he suffered when his car was struck by the of an art gallery, alleging that the defendant
defendant’s truck, allegedly because the defen- charged him a price higher than what was
dant had fallen asleep at the wheel after driving originally quoted to him for the purchase of
all night. At trial, the defendant’s girlfriend a rare sculpture. During the plaintiff’s testi-
testified that she had been with the defendant in mony, he stated that he purchased the sculpture
the truck and had taken over the driving duties from the gallery on a particular date and then
for several hours that night while the defen- realized two days later that his credit card was
dant napped. The plaintiff calls to the stand an charged in an amount over that which he was
acquaintance of the defendant’s girlfriend, to originally quoted by the defendant. During its
testify that the girlfriend told him that she had defense, the defendant presented the testimony
been unable to get out of bed the weekend the of the art gallery’s clerk, who testifies that she
accident occurred because of severe back pain. remembers the plaintiff coming into the gallery
and purchasing the sculpture a week before the
Is the testimony of the acquaintance admis- date testified to by him, because he signed the
sible? purchase order with such an unusual signature.
(A) Yes, for impeachment purposes only. If the plaintiff objects to this testimony,
should the trial court admit it?
(B) Yes, for impeachment purposes and as
substantive evidence as a declaration of (A) No, because the content of the purchase
physical condition. order is hearsay not within any exception.
(C) No, because this means of impeachment (B) No, because the date of purchase is a collat-
can be done only through cross-examina- eral matter.
tion.
(C) Yes, because the purchase order is a past
(D) No, because the plaintiff has not first given recollection recorded.
the girlfriend an opportunity to explain or
deny the statement. (D) Yes, because the clerk’s testimony is
relevant evidence as to the date the sculp-
ture was purchased.
EVIDENCE MULTIPLE CHOICE QUESTIONS 5.
Question 9 Question 10
Following their divorce, the plaintiff and In a wrongful death action for the death of
defendant were engaged in a bitter custody battle his wife in an automobile accident, the plain-
over their two children. The plaintiff is seeking tiff alleged that the accident was caused by a
to testify as to statements made by the defen- mudflap assembly that fell off the defendant’s
dant prior to the divorce in which she had told truck. The plaintiff wishes to introduce the
him that she did not feel that she could properly testimony of a witness, another truck driver who
care for the children by herself. The defendant was on the same highway at the time, who heard
objects, alleging that the statements are privi- someone tell the defendant over CB radio that he
leged as a confidential communication made had noticed at the truck stop that the defendant’s
during the marriage. mudflap assembly on his truck was loose. The
witness does not know the identity of the person
Should the court admit the testimony? who gave the warning.
(A) Yes, because the privilege does not apply. If the defendant objects to admission of the
testimony, the court should rule that it is:
(B) Yes, because the privilege terminates on
divorce. (A) Admissible to prove that the defendant was
notified that the mudflap assembly was
(C) No, because the plaintiff has a motive to lie. loose.
(D) No, because the statements were confiden- (B) Admissible both to prove that the defendant
tial and made during the marriage. was notified that the mudflap assembly was
loose and as substantive evidence that it
was loose.
Question 11 Question 12
The victim collapsed at her desk while The plaintiff brought suit against the defen-
drinking her morning coffee. Her secretary dant for injuries sustained when she was struck
came rushing to her aid. Gasping for breath, the by the defendant’s car while crossing the street.
victim said, “I don’t think I have much time left. The defendant denied liability, claiming that the
I want you to remember when they come looking plaintiff crossed against the light. To establish
for suspects that I believe my assistant would kill liability, the plaintiff offers into evidence the
for my job.” The victim soon lost consciousness. hospital record from her visit to the emergency
She regained consciousness briefly after arriving room, in which she stated that the defendant
at the hospital, but the doctors would not allow failed to come to a complete stop at a red light
her to speak to anyone, including the police. She when the plaintiff entered the crosswalk. The
again lapsed into a coma, and she remains in this physician who made the entry still works at the
vegetative state. It was determined that she was hospital but has not been called to testify.
poisoned. The assistant is arrested and charged
with attempted murder. If the defendant objects to the evidence, the
court should rule that it is:
At the assistant’s trial, the prosecution wishes
to call the victim’s secretary to testify to the (A) Admissible, under the business records
victim’s statement about the assistant before the exception.
ambulance arrived.
(B) Admissible, because the plaintiff is subject
The court should find the statement: to cross-examination as to her statement.
(A) Admissible, because it is a dying declara- (C) Inadmissible, because the emergency room
tion. physician is available to testify.
(B) Admissible, because it is a declaration of (D) Inadmissible, as hearsay not within any
the victim’s state of mind. exception.
Answer to Question 1
(C) The witness can testify to one of the defendant’s statements but not the other. A statement by an
opposing party (i.e., a statement made by a party and offered against that party) is nonhearsay and
thus will not be excluded by the hearsay rule. However, it will be excluded if there is a specific
rule excluding the statement. Federal Rule 408 provides that settlement offers and factual state-
ments made during settlement negotiations are inadmissible if offered to prove or disprove the
validity or amount of a disputed claim. Rule 408 only applies, however, when there is a dispute
between the parties. A statement made at the scene of an accident would rarely qualify. Federal
Rule 409 excludes evidence of the payment or offer to pay medical expenses if offered to help
establish liability for an injury. Rule 409, however, does not exclude factual statements made
in conjunction with the payment or offer. Such factual statements would be admissible as an
opposing party’s statement. Hence, the defendant’s statement would not qualify as a settlement
offer. His statement “I’ll take care of your medical bills” would be excluded under Rule 409. His
statement “I was not paying attention” would not be excluded by Rule 409 and would be admis-
sible as an opposing party’s statement. (A) is wrong because the defendant’s statement is not a
settlement offer. (B) is tempting but wrong. Regardless of whether the plaintiff was negligent per
se, the defendant’s statement would probably be admissible on the issue of comparative fault or the
issue of damages. (D) is too broad a statement. As discussed above, the defendant’s statement “I’ll
take care of your medical bills” would be excluded.
Answer to Question 2
(B) The testimony regarding careful driving is admissible; the testimony regarding truthfulness
is inadmissible. In a criminal case, a defendant may call a qualified witness to testify to the
defendant’s good reputation and opinion for the trait involved in the case. Here, the defendant’s
character for being a reckless driver is directly at issue, and testimony on that issue is admis-
sible. However, the evidence regarding his truthfulness should be excluded because his veracity
for truthfulness is not at issue in a reckless homicide case. Furthermore, a party may not bolster
or accredit the testimony of a witness unless the relevant trait has been attacked. While truthful-
ness is a relevant trait to rehabilitate a witness whose credibility has been impeached, there is
no indication in these facts that the defendant’s credibility was challenged. Thus, the reputation
testimony for truthfulness is inadmissible. (A) and (C) are wrong because, as stated above, the
portion of the testimony relating to the defendant’s reputation as a careful driver is admissible. (A)
and (D) are wrong because, as stated above, testimony relating to the defendant’s truthfulness is
not admissible.
Answer to Question 3
(C) This choice states the critical factor for admitting evidence of other crimes or misconduct to show
motive, while the other choices raise issues that are relevant only when other crimes evidence is
being offered for impeachment purposes. It is essential that you keep the impeachment rules
distinct from the rules for admitting other crimes evidence when the evidence is independently
relevant—the bar examiners will often mix these issues in the answer choices for this type of
question.
One of the most important areas where recurring relevance questions have developed into estab-
lished rules is the use of character evidence. The well-settled rule is that extrinsic evidence of
8. ANSWERS TO MULTIPLE CHOICE QUESTIONS
other crimes is not admissible to show a criminal disposition or conduct in conformity with the
other crimes. On the other hand, Federal Rule 404(b) permits this evidence to be introduced for
other purposes, such as to show motive, opportunity, intent, or identity, whenever these issues
are relevant in the case. Because of the potential for unfair prejudice of this type of evidence, the
balancing test of Federal Rule 403 (paraphrased in choice (C)) is particularly important. Even
though evidence of the other robberies is relevant to show motive, the court may find that its
probative value is substantially outweighed by the danger of unfair prejudice, especially because
the other crimes are of the same type as the crime charged. (A) is incorrect because a conviction
is not required for other crimes evidence used for this purpose. Only when extrinsic evidence of
another crime is being used to impeach a testifying defendant is an actual conviction required.
(B) is incorrect because whether the defendant has testified is relevant only when the other crimes
evidence is being used for impeachment, since a defendant only puts his credibility at issue if he
takes the witness stand; it is irrelevant when the other crimes evidence is used to show motive. (D)
is incorrect. For independently relevant uncharged misconduct by the defendant to be admissible,
there need only be sufficient evidence to support a jury finding that the defendant committed the
prior act; clear and convincing evidence is not required.
Answer to Question 4
(C) Judicial notice operates as a substitute for proof as to facts that are matters of common knowledge
in the community or are capable of certain verification through easily accessible, well-established
sources. When a court takes judicial notice of a fact under the federal rules in a criminal case, the
jury may, but is not required to, accept the fact noticed; thus, its effect is only to relieve the prose-
cutor of her burden of producing evidence on that fact. (A) is incorrect because taking judicial
notice does not affect the burden of persuasion, which is the burden of one litigant to overcome
the case of the opposing litigant. (B) is incorrect because judicial notice of a fact does not estab-
lish proof of the fact beyond a reasonable doubt; as discussed above, the jury is not required to
accept the fact noticed. (D) is incorrect because it is the rule for civil cases; in criminal cases,
the jury is instructed that it may, but is not required to, accept as conclusive any fact judicially
noticed.
Answer to Question 5
(B) The defendant may compel the plaintiff to introduce the remainder of the record. When part of
a writing or recorded statement is introduced into evidence, the Federal Rules provide that the
adverse party may compel the introduction of any other part of the writing or recorded statement
that ought, in fairness, to be considered at the same time. Here, the remainder of the shipping
record that supports the defendant’s position should be introduced at the same time as the part
introduced by the plaintiff. (A) is incorrect because no foundation need be laid for another part
of a record that has already been introduced. (C) is incorrect because Federal Rule 106 does not
require proof of relevance. The standard is fairness. In any case, the remainder of the record
appears to be relevant to the defendant’s claim. (D) is incorrect because, although it is hearsay, a
shipping record from a freight company very likely falls within the business records exception to
the hearsay rule.
Answer to Question 6
(B) The question on cross-examination regarding the witness’s honesty is an appropriate impeachment
method. A witness may be interrogated on cross-examination with respect to any act of miscon-
duct that is probative of truthfulness (i.e., demonstrates dishonesty). Lying on a job application
ANSWERS TO MULTIPLE CHOICE QUESTIONS 9.
would be such an act. Thus, (B) is a proper method of impeaching the witness’s credibility. (A)
is incorrect because juvenile offenses are generally not admissible for impeachment purposes in
civil cases. (C) is incorrect because a conviction cannot be used to impeach a witness if he has
been pardoned based on innocence. (D) is incorrect because a record of an arrest cannot be used
to impeach a witness—an actual conviction is required.
Answer to Question 7
(A) The testimony of the acquaintance is admissible, but only for impeachment purposes. For the
purpose of impeaching the credibility of a witness, a party may show that the witness has, on
another occasion, made statements that are inconsistent with some material part of her present
testimony. Here, the acquaintance is prepared to testify that the girlfriend stated that she had been
unable to get out of bed because of severe back pain at the time that she testified that she was with
the defendant and had taken over the driving, a fact that would be material to the allegations in
the lawsuit. Thus, the testimony is properly admissible for purposes of impeachment. (C) is incor-
rect because, under the Federal Rules, an inconsistent statement may be proved by either cross-
examination or extrinsic evidence. (D) is incorrect because, while extrinsic evidence is admissible
only if the witness is, at some point, given an opportunity to explain or deny the allegedly incon-
sistent statement, the opportunity need not come before the introduction of the statement under
the Federal Rules; the testimony may be admitted now and the girlfriend subsequently be given
an opportunity to explain or deny it. (B) is incorrect because the testimony is hearsay that is
not admissible for substantive purposes because it does not fall under the exception for declara-
tions of physical condition. Under the Federal Rules, declarations of present bodily condition are
admissible as an exception to the hearsay rule when made to anyone, not just a physician, whereas
declarations of past physical condition are admissible as a hearsay exception only if made to
assist in diagnosing or treating the condition. Here, the girlfriend’s statement to the acquaintance
pertained to her past physical condition, and there is no indication that the statement was made to
assist in diagnosis or treatment. Hence, the girlfriend’s statement does not fall within these or any
other exceptions to the hearsay rule, and is admissible for impeachment purposes only.
Answer to Question 8
(B) Testimony as to the date of purchase of the sculpture should not be admitted because its minimal
relevance is substantially outweighed by considerations of waste of time and confusion of the
issues under Rule 403. Whether the plaintiff actually purchased the sculpture one week sooner
than the date testified to by him has no bearing on the amount he should have been charged,
which is the issue in controversy. The only relevance of the clerk’s testimony is to cast doubt on
the plaintiff’s credibility, but it is not admissible for impeachment purposes either. When a witness
makes a statement not directly relevant to the issues in the case, the rule against impeachment
on a collateral matter bars his opponent from proving the statement untrue either by extrinsic
evidence or by a prior inconsistent statement. As noted previously, the plaintiff’s statement as
to the date on which he purchased the sculpture is not directly relevant to any other issue in the
case. Thus, the defendant is not permitted to prove the statement untrue by means of the clerk’s
testimony that the plaintiff made the purchase on a different day. (D) is incorrect even though
it is true that the clerk’s testimony tends to prove that the plaintiff purchased the sculpture on a
date other than that to which he testified. As detailed above, the limited relevance of the date for
impeachment purposes is outweighed by considerations of waste of time and confusion of issues.
(A) is incorrect because the clerk is not testifying to the content of the purchase order. Rather, she
is simply testifying to the date of purchase from her own memory. Thus, the hearsay nature of the
contents of the purchase order is not at issue. (C) is incorrect. Under the past recollection recorded
10. ANSWERS TO MULTIPLE CHOICE QUESTIONS
exception to the hearsay rule, where a witness states that she has insufficient recollection of an
event to enable her to testify fully and accurately, even after she has consulted a writing given to
her on the stand, the writing itself may be introduced into evidence if: (i) the witness at one time
had personal knowledge of the facts recited in the writing; (ii) the writing was timely made when
the matter was fresh in the witness’s mind; (iii) the writing was made by the witness or under her
direction or adopted by her; and (iv) the witness is presently unable to remember the facts. Here,
the clerk has not stated that she has insufficient recollection of the events to which she is testi-
fying, and the defendant is not even attempting to introduce the purchase order into evidence. She
is fully able to testify as to the date on which the plaintiff purchased the sculpture, and is simply
referring to the purchase order because of having seen the plaintiff’s unusual signature on it. Thus,
the purchase order does not constitute a past recollection recorded.
Answer to Question 9
(A) The court should find that the statements are admissible. The privilege for confidential marital
communications provides that a party has a privilege to refuse to disclose, and to prevent another
from disclosing, confidential communications made between spouses during the marriage.
However, the privilege does not apply in actions between the spouses. Because this is an action
between the spouses, the privilege does not apply and the testimony is admissible. Thus, (A) is
correct and (D) is incorrect. (C) is incorrect. The plaintiff would have to be cross-examined as to
any potential bias or motive to lie; the mere possibility of such a motive does not automatically
preclude his testimony. (B) is incorrect because the privilege survives divorce. Here, the state-
ments were made during the marriage and the privilege would apply even though the parties are
now divorced.
Answer to Question 10
(A) The court should rule that the witness’s testimony is admissible nonhearsay for the limited
purpose of showing that the defendant knew that the mudflap assembly was loose. Hearsay is a
statement, other than one made by the declarant while testifying at the current trial or hearing,
offered in evidence to prove the truth of the matter asserted. When the out-of-court statement is
introduced for any other purpose, the statement is not hearsay. Thus, in a negligence case when
knowledge of a danger is an issue, the third person’s statement of warning is admissible for the
limited purpose of showing knowledge or notice on the part of the listener. Here, the statement
of the third person over the CB radio can be testified to by the witness to show that the defendant
was notified of the loose mudflap assembly. (B) is incorrect because the testimony would not be
admissible as substantive evidence that the assembly was loose. In that case, the statement would
be inadmissible hearsay because it was made by an out-of-court declarant and offered to prove
the truth of what the declarant stated. (C) is incorrect because it is not necessary that the out-of-
court declarant be identified. Since the statement is being offered only to show that the defendant
had notice of the loose mudflap assembly, it does not matter that the witness does not know who
made the statement; he can still testify to what he heard spoken to the defendant. (D) is incor-
rect because, as stated above, the statement is admissible nonhearsay for the limited purpose of
proving that the defendant had notice that the mudflap assembly was loose.
Answer to Question 11
(D) The statement is inadmissible because it is hearsay not within any exception. It is an out-of-
court statement being offered for the truth of the matter asserted, i.e., that the victim’s assistant
committed the crime. As will be explained below, the statement does not qualify for exception
ANSWERS TO MULTIPLE CHOICE QUESTIONS 11.
either as a dying declaration or as a declaration of the victim’s state of mind. In addition, the
statement does not constitute an excited utterance. Despite the fact that the declaration was made
while the victim was still under the stress of a startling event, the statement does not qualify
under the exception because it does not concern the immediate facts of the startling occurrence.
The fact that the victim believes that her assistant would kill for her job does not concern what
is happening to her at that moment except to give her opinion of who did the deed. (A) is wrong
for the same reason. The statement does not appear to concern the facts of the cause or circum-
stances of what she believed to be her impending death. A declaration of mere opinion that is not
based on firsthand knowledge is inadmissible. While an argument could be made either way on
this issue, a more certain reason why the statement is inadmissible as a dying declaration is that
the assistant is being tried for attempted murder. Use of dying declarations in criminal prosecu-
tions is limited to homicide cases. (B) is wrong because the state of mind exception covers state-
ments of the declarant’s then-existing state of mind, emotion, sensation, or physical condition,
and is applicable only to show the declarant’s state of mind when it is directly in issue or to show
subsequent acts of the declarant. Neither of these situations is present here. Thus, this exception
is inapplicable. (C) is incorrect. Under Federal Rule 803(3), a declaration of present bodily condi-
tion is admissible as an exception to the hearsay rule if it relates to physical symptoms. Under
Rule 803(4), a declaration of a past bodily condition, including a statement as to the cause of the
condition, is admissible if it is made to assist in diagnosis or treatment. While her statement that
she believed she was dying might be admissible as a declaration of present bodily condition, her
statement that her assistant would kill her for her job is neither describing her symptoms nor made
to assist in diagnosis or treatment.
Answer to Question 12
(D) The court should rule that the statement is inadmissible hearsay. Hearsay is an out-of-court state-
ment offered to prove the truth of the matter asserted, and is inadmissible unless it falls within
an exception to the hearsay rule. Hearsay within hearsay refers to an out-of-court statement that
contains other hearsay, and is admissible only if both the outer hearsay and the inner hearsay
fall within an exception to the hearsay rule. Here, the hospital record itself is hearsay, and the
plaintiff’s statement contained within the record is also hearsay. Therefore, both the record and
the statement contained within the record must fall within an exception to the hearsay rule to
be admissible. Neither the hospital record nor the statement within it falls within any exception
to the hearsay rule; hence, the record is inadmissible. (A) is incorrect because a hospital record
can qualify as a business record only to the extent that it is related to the medical diagnosis or
treatment of a plaintiff. Here, the statement that the defendant did not stop at the red light is
not related to the plaintiff’s treatment, and is therefore inadmissible under the business records
exception. (B) is incorrect. Prior declarations of a witness on the stand may be hearsay because
they were not subject to contemporaneous cross-examination. Here, the plaintiff’s statement in
the hospital record was not subject to cross-examination, and the plaintiff is offering that state-
ment as evidence rather than testifying at trial as to what happened. (C) is incorrect because it
only addresses one of the hearsay issues. The testimony would still be inadmissible hearsay if the
physician testified because the physician would be testifying as to the plaintiff’s statement, which
does not fall within any exception to the hearsay rule.
APPROACH TO EVIDENCE 1.
APPROACH TO EXAMS
EVIDENCE
IN A NUTSHELL: The law of evidence is a system of rules and standards governing the admissibil-
ity of proof (testimony, writings, objects, etc.) at the trial of a civil or criminal action. A good rule of
thumb is that all relevant evidence is admissible if competent, i.e., does not violate some exclusionary
rule. These exclusionary rules serve as a gateway, and are meant to ensure that evidence is reliable
and authentic (e.g., the hearsay rule, the best evidence rule, Dead Man Statutes), or to protect certain
societal interests outside of the courtroom (e.g., testimonial privileges).
2. Allows the trier of fact to reach conclusions based upon its own perceptions
rather than relying upon those of witnesses
B. Types and Examples
1. Direct
a. Offered to prove the facts about the object as an end in itself
b. Example—evidence of a permanent injury can be shown by the injury
itself
2. Circumstantial
a. Facts about the object are proved as a basis for an inference that other
facts are true
b. Example—in a paternity case, the child can be shown to the jury to
demonstrate the child is the same race as the alleged father
3. Original
a. Has some connection with the transaction that is in question at the trial
b. Example—the alleged murder weapon in a murder case
4. Prepared
a. Also called “demonstrative” evidence
b. Examples—sketches, models, jury view of scene
C. Must Be Authenticated
1. Recognition testimony by witness—if item has recognizable features
2. Chain of custody—if evidence is a type likely to be confused or if it can be
easily tampered with:
a. Proponent must show that the object has been held in a substantially
unbroken chain of possession
b. Need not negate all possibilities of substitution or tampering, but must
show adherence to some system of identification and custody
III. DOCUMENTARY EVIDENCE—COMMON ISSUES
A. Has It Been Authenticated?
1. Writings must be authenticated by proof showing they are what the proponent
claims they are (unless self-authenticating such as newspapers, commercial
paper, etc.)
2. Authenticated by stipulation, evidence of authenticity, etc.
3. Oral statements—only when important
B. Best Evidence Rule (“Original Document Rule”)
1. In proving the terms of a writing (including a recording, photograph, or X-ray),
where the terms are material, the original writing must be produced
a. Secondary evidence is allowed only if the original is unavailable for
some reason other than the serious misconduct of the proponent
2. Rule applies only in certain situations
a. When the writing is a legally operative or dispositive instrument (e.g.,
contract, deed, will, divorce decree)
b. When the witness’s knowledge of a fact comes from having read it in
the document
1) If the fact exists independently of a writing, the best evidence
rule does not apply
4. APPROACH TO EVIDENCE
The essay questions that follow have been selected to provide you with an opportunity to experience
how the substantive law you have been reviewing may be tested in the hypothetical essay examination
question context. These sample essay questions are a valuable self-diagnostic tool designed to enable
you to enhance your issue-spotting ability and practice your exam writing skills.
It is suggested that you approach each question as though under actual examination conditions. The
time allowed for each question is 45 minutes. You should spend 10-15 minutes spotting issues, under-
lining key facts and phrases, jotting notes in the margins, and outlining your answer. If you organize
your thoughts well, about 30 minutes will be more than adequate for writing them down. Should you
prefer to forgo the actual writing involved on these questions, be sure to give yourself no more time for
issue-spotting than you would on the actual examination.
The BARBRI technique for writing a well-organized essay answer is to (i) spot the issues in a
question and then (ii) analyze and discuss each issue using the “CIRAC” method:
C — State your conclusion first. (In other words, you must think through your answer before you
start writing.)
I — State the issue involved.
R — Give the rule(s) of law involved.
A — Apply the rule(s) of law to the facts.
C — Finally, restate your conclusion.
After completing (or outlining) your own analysis of each question, compare it with the BARBRI
model answer provided herein. A passing answer does not have to match the model one, but it should
cover most of the issues presented and the law discussed and should apply the law to the facts of the
question. Use of the CIRAC method results in the best answer you can write.
2. EVIDENCE EXAM QUESTIONS
After performing routine liposuction surgery on Bob Boyd, Dr. Ann Adams was distressed to
discover that the surgery did not achieve the anticipated result. In fact, there were unforeseen compli-
cations that resulted in emergency surgery to remove a blood clot. Fortunately, Boyd made a full
recovery, but not before enduring a 10-day hospital stay, incurring significant medical bills, and
missing two weeks of work.
After the surgery and during Boyd’s recuperation, Dr. Adams, who is a very conscientious, compas-
sionate physician, continually met with Boyd and his family; on at least two of these occasions she
expressed her sympathy for the situation and her regret that the surgery had not yielded the anticipated
result.
Dr. Adams, who remained haunted by Boyd’s surgery, spoke on several occasions with her husband
about her concern that she may have in some way contributed to Boyd’s surgical problems as she had
performed the surgery early in the morning, following a night of partying, when she had a severe
headache, possibly the result of a hangover. Her husband attempted to reassure Dr. Adams, telling her
that all doctors make mistakes and advising her that under no circumstances should she admit any
liability. She similarly confided her concerns to her personal physician, whom she consulted when she
continued to suffer from severe headaches.
Prior to Boyd’s full recovery, Dr. Adams met with her accountant to discuss how she might set up
some sort of trust or annuity for Boyd’s family in the event he did not recover. Because Boyd fortu-
nately did make a full recovery, Dr. Adams did not pursue this idea any further.
After his recovery, Boyd filed a medical malpractice suit against Dr. Adams in Clarke County
superior court, alleging that she was negligent in performing the liposuction surgery that resulted in his
complications. Among other things, he sought loss of income for the two weeks of work he missed.
Upon being served with the lawsuit, Dr. Adams put her malpractice carrier on notice. Dr. Adams
then called her personal lawyer, Lou Lawyer, who agreed to meet with her the very next day, on New
Year’s Day. Dr. Adams, who was visibly upset about the lawsuit, took her sister Karen (who has always
been her best friend and confidante), with her to sit in on the meeting to provide moral support.
The insurance carrier has retained you to represent Dr. Adams in the medical malpractice litiga-
tion. You receive a call from Lou Lawyer, who tells you that she is Dr. Adams’s personal lawyer and
that she, Lawyer, will represent Dr. Adams’s personal interests because the interests of Dr. Adams
and her malpractice carrier may not always coincide. When you meet with Dr. Adams, she confides
her concerns about having conducted the surgery while suffering from a severe headache. Before
Dr. Adams leaves your office, she entrusts to your safekeeping her entire personal file on Boyd and
his surgery, which contains Boyd’s medical charts and related information created on the day of the
surgery.
Prior to the date of trial, Boyd’s counsel sends you the plaintiff’s portion of the pretrial order in
which he is required to identify all witnesses, with a brief summary of their anticipated testimony. In
this pleading, Boyd’s counsel indicates that he intends to call the following witnesses at trial:
1. Dr. Adams’s physician: Any statements regarding the Boyd surgery which Dr.
Adams may have made to her physician.
2. Dr. Adams’s husband: Any statements regarding the Boyd surgery which Dr.
Adams may have made to her husband.
3. Dr. Adams’s accountant: Any statements regarding the Boyd surgery which
Dr. Adams may have made to her accountant.
EVIDENCE EXAM QUESTIONS 3.
4. Karen: Any statements regarding the Boyd surgery which Dr. Adams may
have made to Lou Lawyer during the New Year’s Day conference and any advice
which Lou Lawyer may have given Dr. Adams during that conference.
5. Dr. Adams:
(b) Any statements of sympathy, regret, or like statements which Dr. Adams may
have made to Boyd or his family after the surgery.
You now must prepare your portion of the pretrial order in which you are required to indicate what
objections, if any, you have to each witness identified and the proposed subject(s) of testimony.
(1) For each witness listed above, please specify what objections, if any, should be made. In each
instance, explain your reason(s) for each objection, or your reason(s) for determining that no objection
is appropriate. Please direct your response only to the indicated subject matter of testimony.
(2) Assume that you indicate in your portion of the pretrial order that you intend to introduce
evidence of Boyd’s insurance policies that provide him with compensation for loss of work (income
replacement) in this situation. What objections, if any, do you anticipate from your opposing counsel?
4. EVIDENCE EXAM QUESTIONS
Big, Inc. (“Big”) has its corporate headquarters exactly in the middle of a park owned and
maintained by Big. The park has an eight-foot-wide asphalt walkway weaving in and out among the
trees, flower gardens, park benches, and statues on the park grounds. Big has known that the public
uses the walkway and park extensively, and Big has never objected.
One of the trees growing close to the walkway to the south of the corporate building has a tree root
under the walkway. Over the years, the root caused the surface of the asphalt to rise up about three
inches at the walkway edge nearest the tree and about two inches at the walkway edge farthest from
the tree. Kelly has a habit of making at least three circuits of the walkway every day between noon and
1 p.m. according to Pat, who has habitually eaten lunch on a particular park bench every day. This park
bench was located to the north of the corporate building. Kelly and Pat are acquainted with each other
because of an old and bitter lawsuit between them.
Kelly stumbled over the raised asphalt and fell. The corporate building was located between the
bench where Pat sat and the place where Kelly fell, making it impossible for Pat to see Kelly fall. An
unknown person called for an ambulance to take Kelly to the local hospital. After the fall, Big removed
the tree root and resurfaced that part of the walkway. Also, Big paid for Kelly’s ambulance and hospital
bills.
After six months, Big received a letter from Kelly’s attorney, which claimed that Kelly received
a serious knee injury and made a demand for $500,000. The insurance adjuster for Big’s insurance
company tried to negotiate a settlement with Kelly’s attorney both by phone calls and letters, but was
unsuccessful. Kelly filed suit against Big in a jurisdiction that follows the Federal Rules of Evidence. In
its answer, Big admitted that it owned and controlled the park and the walkway. Answer the following
questions:
(1) At the trial, Kelly’s attorney wants to introduce evidence that Big paid for the medical bills to
show that Big is liable for Kelly’s injuries. Big’s attorney objects. How should the judge rule and why?
(2) At the trial, Kelly’s attorney wants to introduce evidence that Big removed the tree root and
resurfaced the walkway after Kelly’s fall:
(a) To show that it was a dangerous condition. Big’s attorney objects. How should the judge rule
and why?
(b) To show that Big owned and controlled the park and walkway surrounding the corporate
headquarters. Big’s attorney objects. How should the judge rule and why?
(3) At the trial, Kelly’s attorney wants to introduce evidence that Big had made several offers to
settle Kelly’s claim. Big’s attorney objects. How should the judge rule and why?
(4) At the trial, Big’s attorney wants to introduce evidence that Kelly has a habit of walking over
the walkway at least three times a day to show that Kelly knew or should have known of the existence
of the raised surface of the asphalt at the spot where Kelly fell. Kelly’s attorney objects. How should
the judge rule and why?
(5) At the trial, Big’s attorney wants to introduce evidence about how unreasonable and unkind
Kelly was about the events leading up to the old, bitter lawsuit between Pat and Kelly. Kelly’s attorney
objects. How should the judge rule and why?
EVIDENCE EXAM QUESTIONS 5.
Johnson, a longtime farmer, had grown beans for more than 20 years. His large operation, during an
average year, yielded annual returns of more than $350,000. The 700 acres that he maintained used all
of the latest techniques for fertilizing, planting, and harvesting the crops.
The neighboring farm was operated by the Harolds. They had been in business less than five years
and believed cotton grew best on that type of land. However, their inexperience made their farming
operations marginal at best.
The Harolds solicited the services of Plane-O, an out-of-state crop dusting company. Plane-O’s
aircraft was fitted with a unique video camera that would actually film crops as they were being
sprayed. Using several of his best pilots, Plane-O sprayed the Harolds’ farm for an entire week. They
did so following the strict instructions and mapping directions of the Harolds. But less than two days
after being given their instructions, the pilots followed their own maps and surveys.
During an inspection of his fields in April of that year, Johnson noticed unusual leaf and stem
damage to his beans. He believed that the damage to his crops was caused by the spraying residue
that had been blown from his neighbor’s fields. The nearest agricultural agent lived 50 miles away and
would not come to inspect the crops. Johnson had a grandson who was visiting for the summer. The
grandson took several pictures of the plants. These were mailed to the agricultural agent. The agent
thought the photographs of the bean field were just like any other pictures. Weeks later, the damage
had grown worse, with 80% of the beans affected.
One year later, Johnson has sued the Harolds and Plane-O for his crop loss. Plane-O has refused
to come to the state and allow its airplane film or its pilots to be used. They want to settle their claim
out of court. Johnson wants the agricultural agent to testify about the crops by using the airplane film
and the photographs. The judge has requested a pretrial memorandum as to the evidence in this case.
Relying on the Federal Rules of Evidence, please prepare the document for presentation to the court.
6. EVIDENCE EXAM QUESTIONS
Ann was driving towards a stoplight. As she entered the intersection, she was struck from the right
by Dan Defendant. Riding in the front seat with Ann was Sally Plaintiff.
After the impact, Wilbert came up to Ann’s car to ask if the two ladies were all right. Ann looked at
Wilbert and said, “But, the other guy (Dan) ran the light.” Wilbert looked back and said, “Yes, I know,
because I saw him run the light, also.”
George, Wilbert’s friend, came up to the car just in time to hear Wilbert make his statement. Wilbert
left to go check on Dan while George stayed to talk to the ladies. George told the ladies that he was
looking away until after he heard the impact, and he did not see the light. George watched Ann calm
Sally down and heard Ann say three or four times, “You’ll be OK, Sally, he ran the light and we can
sue him.” George also heard Sally moan, “My knees hurt so bad!”
In a few minutes Wilbert came back and told Ann, Sally, and George that Dan had told him
(Wilbert) that he (Dan) was talking on his cell phone and because he was not looking at the light as
he approached the intersection, he (Dan) had no idea what color the light was. Wilbert also said that
Dan said that he (Dan) was driving home after celebrating with friends at the Beer Bust Bar. A blood
alcohol test did not reveal any alcohol in Dan’s blood.
When the police came to investigate, Dan told the police that his light was green and that there was
nothing distracting him as he approached the light. This is what the police put in the police report. The
police also included George as a witness but failed to include Wilbert as a witness.
At the hospital, Sally told the triage nurse that her neck, back, and knees were hurting. This was put
into the medical records, resulting in diagnostic procedures and treatment for Sally’s neck, back, and
knees.
(1) Sally sues Dan for personal injuries in a jurisdiction that follows the Federal Rules of Evidence.
Sally wants to have George and Wilbert testify about Dan’s celebration at the Beer Bust Bar. Dan
objects. State whether or not the objection will succeed with each witness and why.
(2) Sally wants to have George and Wilbert testify that Dan said he was talking on his cell phone
and was not looking at the light as Dan approached the intersection. Dan objects. State whether or not
the objection will succeed with each witness and why.
(3) Sally wants to have George and Wilbert testify that Dan ran the red light. Dan objects. State
whether or not the objection will succeed with each witness and why.
(4) In her deposition, Sally said that immediately after the impact her neck, back, and knees were
hurting badly. Dan wants to have Ann and George testify that Sally did not complain of pain in her
neck and back at the scene of the impact. Sally objects. State whether or not the objection will succeed
with each witness and why.
(5) In her lawsuit, Sally sues for injuries to her ankles. Dan wants to introduce the triage nurse’s
notes to show that there was no complaint of ankle injury to the triage nurse. Sally objects. State
whether or not the objection will succeed and why.
EVIDENCE EXAM ANSWERS 1.
(1) Hearsay would not be an appropriate objection to any testimony regarding Dr. Adams’s
statements. The issue is whether any of the offered testimony is hearsay or otherwise inadmissible.
Hearsay is an out-of-court statement offered to prove the truth of the matter asserted. Statements by
an opposing party are admissible nonhearsay. Thus, any statements made by Dr. Adams are admis-
sible.
Dr. Adams’s physician: No objection should be made to the testimony by Dr. Adams’s physi-
cian. The physician-patient privilege prohibits physicians from releasing any medical informa-
tion concerning a patient. Here, however, statements Dr. Adams made regarding the surgery she
performed on Boyd would probably not be considered “medical information” and, therefore, would be
admissible.
Dr. Adams’s husband: An objection to the testimony of Dr. Adams’s husband should be made
based on the privilege for confidential marital communications. Communications between spouses are
excluded on public policy grounds. For this privilege to apply, the communication must be made during
a valid marriage, and the communication must be made in reliance upon the intimacy of the marital
relationship. Here, Dr. and Mr. Adams are married, and Dr. Adams expressed her concerns about Boyd
to her husband in reliance upon the intimacy of their relationship. Therefore, these statements are privi-
leged and inadmissible.
Dr. Adams’s accountant: An objection to the testimony of Dr. Adams’s accountant should be made
based on the accountant-client privilege if the jurisdiction recognizes this privilege. Communica-
tions between an accountant and a client are privileged if they are made in a professional capacity
and are within the scope of the representation. Here, Dr. Adams talked to her accountant about poten-
tially setting up a trust or annuity for Boyd’s family. She met with the accountant for the purpose of
discussing this, so the communications were made in a professional capacity. Setting up a trust or
annuity is also within the scope of the accountant’s representation of Dr. Adams, and a court would
likely find that explaining why she wants to set up such a trust or annuity is also within the scope.
Dr. Adams’s failure to pursue the matter does not destroy the privilege. Therefore, any statements
Dr. Adams made to her accountant regarding Boyd are inadmissible if the jurisdiction recognizes the
accountant-client privilege.
Karen: No objection should be made to Karen testifying about statements Dr. Adams made to
Lou Lawyer. Confidential communications between an attorney and client, made during professional
consultation, are privileged from disclosure. Communications are “confidential” if they were not
intended to be disclosed to third persons, other than those to whom disclosure would be in furtherance
of the rendition of legal services to the client or those who are necessary for the transmission of the
communication (e.g., an interpreter). Communications made in the known presence and hearing of any
other third person are not privileged. Here, although Karen attended Dr. Adams’s conference with Lou
Lawyer to provide moral support, her presence was not necessary to the rendition of legal services or
the transmission of communications. Thus, Karen may testify to Dr. Adams’s statements made during
the New Year’s Day conference.
However, an objection to Karen’s testimony about any advice which Lawyer may have given during
the conference should be made based on hearsay. As discussed above, hearsay is generally inadmissible
unless it falls under one of the exceptions to the hearsay rule. Here, Lawyer’s unprivileged communica-
tions do not fall under any such exceptions and, therefore, are inadmissible.
Dr. Adams: (a) An objection that any prior malpractice claims against Dr. Adams are irrelevant
should be made. Evidence of similar occurrences involving the same instrumentality is only admissible
if those occurrences happened under the same or similar circumstances, are probative of the material
issue involved, and have a probative value that outweighs the risk that the evidence will confuse the
2. EVIDENCE EXAM ANSWERS
jury or result in unfair prejudice. Here, the facts do not indicate what the other malpractice claims are
about, so it is unclear whether they occurred under the same or similar circumstances. If they did not
occur under similar circumstances, they are irrelevant and inadmissible. However, if the court finds
that the prior claims are similar, then an objection should be made that the probative value of any prior
malpractice claims against Dr. Adams are substantially outweighed by their prejudicial effect. Here,
because evidence of other similar malpractice claims could cause the jury to confuse them with the
Boyd claim, it should be excluded.
(b) No objection should be made regarding the statements made by Dr. Adams to Boyd and his
family. Statements by an opposing party (i.e., statements made by a party and offered against that
party) are admissible nonhearsay. Here, the statements made by Dr. Adams regarding her regret that
the surgery did not achieve the anticipated result have been offered against her at trial and are therefore
admissible as an opposing party’s statements.
(c) An objection that the existence and extent of Dr. Adams’s malpractice insurance is inadmis-
sible should be made. Evidence that a person was or was not insured against liability is not admissible
to show that she acted negligently or is able to pay a substantial judgment. Therefore, opposing counsel
cannot ask Dr. Adams about her malpractice insurance.
(d) No objection should be made to Dr. Adams’s testimony regarding the contents of her file on
Boyd and his surgery. The file is admissible under the business records exception to the hearsay rule,
which allows admission of any writing or record made in the regular course of business as proof of
the recorded act, transaction, occurrence, or event. Note that the physician-patient privilege would not
prohibit Dr. Adams’s testimony because Boyd would likely provide his written authorization.
(2) Opposing counsel will likely object to the introduction of evidence of Boyd’s insurance policies
under the collateral source rule. The collateral source rule states that damages are not reduced or
mitigated by benefits that a plaintiff receives from another source, such as insurance. Defendants may
not introduce evidence relating to any such financial aid from other sources. Therefore, Dr. Adams will
not be able to introduce evidence that Boyd’s insurance will compensate him for loss of work because
his insurance is a collateral source and cannot be considered.
(1) The judge should sustain Big’s objection. As a general rule, evidence is admissible if it is
relevant (i.e., it has a tendency to prove or disprove a material issue), its probative value is not substan-
tially outweighed by unfair prejudice or confusion, and admission would not violate public policy.
Evidence that a party paid (or offered to pay) the injured party’s medical expenses is not admissible to
prove liability for the injury. This rule is based on the public policy concern that such payment might
be prompted solely by “humanitarian motives.” Therefore, the evidence that Big paid Kelly’s medical
expenses is inadmissible.
(2)(a) The judge should sustain Big’s objection. Big’s actions after Kelly’s fall (i.e., removing the
tree root and resurfacing the walkway) constitute subsequent remedial measures. As a matter of public
policy, subsequent remedial measures are inadmissible to prove negligence or culpable conduct. Here,
Kelly is introducing the evidence to show that a dangerous condition existed, which is essential to her
negligence claim against Big. Therefore, the subsequent remedial measures are inadmissible.
(b) The judge should sustain Big’s objection. Subsequent remedial measures are admissible for
certain purposes other than to prove culpability, such as to show ownership and control. However, Big
already admitted in its answer that it owned and controlled the park and walkway, so that is not an
issue in dispute. Therefore, the subsequent remedial measures should not be admitted.
(3) The judge’s ruling will depend on the purpose for which Kelly is offering the evidence of Big’s
settlement offers. In civil cases, per public policy, evidence of offers to compromise is inadmissible
EVIDENCE EXAM ANSWERS 3.
to prove or disprove the validity or amount of a disputed claim. However, such evidence is not neces-
sarily inadmissible when offered for another purpose, such as proving bias or prejudice of a witness,
responding to a contention of undue delay, or proving an effort to obstruct a criminal investigation or
prosecution. It is likely that Kelly is offering this evidence to show that Big is liable; however, the facts
do not specifically indicate that this is the case. Therefore, Big’s objection certainly will be sustained
if Kelly is introducing the settlement offers to prove liability, but it may be overruled if Kelly is intro-
ducing them for another purpose.
(4) The judge should overrule Kelly’s objection. As noted above, evidence is admissible only if
relevant. Evidence is relevant if it tends to make the existence of any fact that is of consequence to
the determination of an action more probable or less probable than it would be without the evidence.
Often, evidence of similar prior acts is not relevant. However, here, evidence of Kelly’s frequent walks
is relevant to prove her knowledge of the defect in the walkway, which is at issue in the case. (Habit
evidence also is admissible to prove that a person’s conduct on an occasion was in conformity with that
habit; however, it is uncontroverted that Kelly was walking on the walkway that day.) Therefore, the
judge should admit the evidence of Kelly’s frequent walks.
(5) The judge should sustain Kelly’s objection. Evidence of character to prove the conduct of a
person in the litigated event is generally not admissible in a civil case. The reasons given are that the
slight probative value of character is outweighed by the danger of prejudice, the possible distraction
of the jury from the main question in issue, and the possible waste of time required by examination of
collateral issues. Kelly’s conduct regarding a prior lawsuit is utterly irrelevant and immaterial in the
present lawsuit.
The evidence in this case consists of the agricultural agent’s testimony based on the airplane film
and the grandson’s photographs. Under the Federal Rules of Evidence, an expert may testify if the
subject matter is one where scientific, technical, or other specialized knowledge will assist the trier
of fact in understanding the evidence or determining a fact in issue. In this case, if the agricultural
agent’s testimony will concern the cause and extent of the bean field damage, one can argue that it
is scientific/specialized knowledge that will assist the trier of fact in making a determination as to
the Harolds’ and Plane-O’s liability. Thus, the agricultural agent would be permitted to testify as an
expert.
Here, Johnson wants the agricultural agent to base her opinion on the airplane film and his grand-
son’s photographs. An expert may base her opinion on facts inadmissible in evidence if the facts are
of a type reasonably relied upon by experts in the particular field. Therefore, the agricultural agent
could base an opinion on the airplane film and the photographs and not admit them into evidence if
other experts in the field would rely upon them in basing an opinion. Although the facts indicate that
Plane-O refuses to allow its film to be used, if Plane-O is subject to the court’s jurisdiction, Johnson
can obtain the airplane film through discovery. The agricultural agent could then examine the airplane
film and the photographs and provide testimony in court.
The airplane film and photographs also could be admitted into evidence if authenticated. Film/
photographs are authenticated if identified by a witness as a portrayal of certain facts relevant to the
issue and verified by the witness as a correct representation of those facts. The witness need not be the
photographer so long as he is familiar with the scene depicted. Consequently, in order for the photo-
graphs to be admitted in evidence, Johnson or his grandson would have to testify. In order for the
airplane film to be admitted, one of the pilots would probably have to testify. If Plane-O is subject to
the court’s jurisdiction, his pilots could possibly be subpoenaed to testify about the film. An argument
might be made that Johnson could authenticate the film. However, he would have to be familiar with
all the scenes depicted in the film, and not just his bean field.
4. EVIDENCE EXAM ANSWERS
(1) The objection will be sustained as to both George and Wilbert because the evidence is irrel-
evant. Relevant evidence is evidence that has a tendency to make the existence of any fact that is of
consequence more or less probable than it would be without the evidence. Here, evidence that Dan was
at the Beer Bust Bar prior to the accident is irrelevant. Such a proposition might be relevant to prove
that Dan had been drinking alcohol prior to the accident; however, the blood alcohol test that was
performed did not reveal any alcohol in Dan’s blood. Thus, neither George nor Wilbert can be called to
testify that Dan was celebrating at the Beer Bust Bar. [Note that if the evidence was relevant for other
purposes, Wilbert could testify as to Dan’s statement about his celebration because it is an opposing
party’s statement, but George could not because it is hearsay within hearsay.]
(2) The objection will be overruled as to Wilbert, but will be sustained as to George.
Wilbert can testify as to Dan’s statement because it is an opposing party’s statement, which is admis-
sible nonhearsay. A statement made by a party and offered against that party is not hearsay. Here, Dan
told Wilbur that he was talking on his cell phone and was not looking at the light. Dan’s statement is
now being offered against him and concerns a relevant fact in the case, i.e., the color of the light. Thus,
the statement is admissible as an opposing party’s statement.
George cannot testify as to what Dan told Wilbert because the statement is hearsay within hearsay.
Hearsay within hearsay is admissible only if both the outer hearsay and the inner hearsay statements
fall within an exception to the hearsay rule. As stated above, the statement from Dan to Wilbert is
admissible. However, the statement from Wilbert to George recounting Dan’s statement does not fall
within any hearsay exception. Thus, George cannot testify as to Dan’s statement to Wilbert.
(3) The objection will be overruled as to Wilbert, but will be sustained as to George.
Wilbert can testify as to whether Dan ran the red light because the evidence is relevant and because
he is a competent witness. As stated above, relevant evidence is evidence that has a tendency to make
the existence of any fact that is of consequence more or less probable than it would be without the
evidence. Here, the evidence is relevant to prove Dan’s liability. A witness is competent if he has
personal knowledge of the matter and he declares that he will testify truthfully. Wilbert is competent
because he has personal knowledge as to whether Dan ran the red light—he saw the accident happen.
Additionally, if he testifies he will be under oath to tell the truth. Thus, the objection will be overruled
as to Wilbert.
George cannot testify as to whether Dan ran the red light because he did not see the accident happen
and therefore has no personal knowledge. He is relying on the statement made by Wilbert, which is
hearsay not within any exception. Thus, the objection will be sustained as to George.
(4) The objection will be overruled as to both Ann and George. The purpose of the testimony
would be to impeach Sally’s credibility as a witness. To impeach a witness, a party can show that the
witness has previously made statements that are inconsistent with some material part of her testimony.
Here, Ann and George can both be questioned as to whether Sally failed to complain of pain in her
back and neck at the scene of the accident. If Sally testifies in accordance with her deposition testi-
mony (that her back and neck did hurt), the testimony of Ann and George can be used to impeach her.
(5) The objection will be overruled. The triage nurse’s notes are admissible under the business
records exception to the hearsay rule. Any writing or record made as a memorandum or record of
any event is admissible in evidence if made in the regular course of any business. Entries in hospital
records qualify under this exception to the extent they are related to the medical diagnosis or treatment
of the patient. Here, the notes are admissible because they were made by the nurse in the ordinary
course of her business—providing patients with medical care. The document can be authenticated by
having the custodian or other qualified witness testify to the identity of the record or by certifying in
writing that the document meets the requirements of the business records exception. Thus, the objec-
tion will be overruled and the notes are admissible.